User login
Sacral blistering
This patient had sustained multiple pressure injuries. The superior aspect of this image shows bullous change with intact dermis, which would classify that area of injury as a Stage 2 pressure injury.1 An older injury in the coccygeal area was through the dermis (Stage 3), with some eschar seen at the base, making that area unstageable.1 (There may have been deeper injury under the eschar.)
This patient was at heightened risk for pressure injury because of his paraplegia.2 Fortunately, he had some preserved sensation. However, his rotator cuff surgery made it harder for him to smoothly transfer to and from the wheelchair, leading to sheer forces against his skin. Social determinates of health care pose an additional risk for pressure injuries. Without a properly fitted wheelchair and cushion, there is an increased risk of localized pressure over both bony prominences and parts of the body that come into contact with mechanical elements of the wheelchair.
Treatment for all pressure injuries includes relief of pressure on the affected area. In this patient’s case, he had to stay in bed (and out of the wheelchair) so that he could heal.
This patient was very knowledgeable about his condition and pressure injuries. He had already arranged for a wheelchair fitting and a visit with the wound care team. His Stage 2 injury had a bullous change instead of absent epithelium, so rather than an adherent hydrocolloid dressing (which would likely remove the loosened epithelium), he was provided with a nonadherent dressing to the area, then an absorbent foam overdressing.
An image of the patient’s deeper sacral injury was shared with the wound care team, which recommended filling the area with a silver rope dressing for its absorptive filler and antibacterial properties. The area was then covered with an absorbent foam. The patient planned to follow up with the Wound Care Clinic for reevaluation and ongoing treatment.
Images and text courtesy of Daniel Stulberg, MD, FAAFP, Professor and Chair, Department of Family and Community Medicine, Western Michigan University Homer Stryker, MD School of Medicine Kalamazoo.
1. 2019 Guideline. The National Pressure Injury Advisory Panel. Accessed October 9, 2022. https://npiap.com/page/2019Guideline
2. Ricci JA, Bayer LR, Orgill DP. Evidence-based medicine: the evaluation and treatment of pressure injuries. Plast Reconstr Surg. 2017;139:275e-286e. doi: 10.1097/PRS.0000000000002850
This patient had sustained multiple pressure injuries. The superior aspect of this image shows bullous change with intact dermis, which would classify that area of injury as a Stage 2 pressure injury.1 An older injury in the coccygeal area was through the dermis (Stage 3), with some eschar seen at the base, making that area unstageable.1 (There may have been deeper injury under the eschar.)
This patient was at heightened risk for pressure injury because of his paraplegia.2 Fortunately, he had some preserved sensation. However, his rotator cuff surgery made it harder for him to smoothly transfer to and from the wheelchair, leading to sheer forces against his skin. Social determinates of health care pose an additional risk for pressure injuries. Without a properly fitted wheelchair and cushion, there is an increased risk of localized pressure over both bony prominences and parts of the body that come into contact with mechanical elements of the wheelchair.
Treatment for all pressure injuries includes relief of pressure on the affected area. In this patient’s case, he had to stay in bed (and out of the wheelchair) so that he could heal.
This patient was very knowledgeable about his condition and pressure injuries. He had already arranged for a wheelchair fitting and a visit with the wound care team. His Stage 2 injury had a bullous change instead of absent epithelium, so rather than an adherent hydrocolloid dressing (which would likely remove the loosened epithelium), he was provided with a nonadherent dressing to the area, then an absorbent foam overdressing.
An image of the patient’s deeper sacral injury was shared with the wound care team, which recommended filling the area with a silver rope dressing for its absorptive filler and antibacterial properties. The area was then covered with an absorbent foam. The patient planned to follow up with the Wound Care Clinic for reevaluation and ongoing treatment.
Images and text courtesy of Daniel Stulberg, MD, FAAFP, Professor and Chair, Department of Family and Community Medicine, Western Michigan University Homer Stryker, MD School of Medicine Kalamazoo.
This patient had sustained multiple pressure injuries. The superior aspect of this image shows bullous change with intact dermis, which would classify that area of injury as a Stage 2 pressure injury.1 An older injury in the coccygeal area was through the dermis (Stage 3), with some eschar seen at the base, making that area unstageable.1 (There may have been deeper injury under the eschar.)
This patient was at heightened risk for pressure injury because of his paraplegia.2 Fortunately, he had some preserved sensation. However, his rotator cuff surgery made it harder for him to smoothly transfer to and from the wheelchair, leading to sheer forces against his skin. Social determinates of health care pose an additional risk for pressure injuries. Without a properly fitted wheelchair and cushion, there is an increased risk of localized pressure over both bony prominences and parts of the body that come into contact with mechanical elements of the wheelchair.
Treatment for all pressure injuries includes relief of pressure on the affected area. In this patient’s case, he had to stay in bed (and out of the wheelchair) so that he could heal.
This patient was very knowledgeable about his condition and pressure injuries. He had already arranged for a wheelchair fitting and a visit with the wound care team. His Stage 2 injury had a bullous change instead of absent epithelium, so rather than an adherent hydrocolloid dressing (which would likely remove the loosened epithelium), he was provided with a nonadherent dressing to the area, then an absorbent foam overdressing.
An image of the patient’s deeper sacral injury was shared with the wound care team, which recommended filling the area with a silver rope dressing for its absorptive filler and antibacterial properties. The area was then covered with an absorbent foam. The patient planned to follow up with the Wound Care Clinic for reevaluation and ongoing treatment.
Images and text courtesy of Daniel Stulberg, MD, FAAFP, Professor and Chair, Department of Family and Community Medicine, Western Michigan University Homer Stryker, MD School of Medicine Kalamazoo.
1. 2019 Guideline. The National Pressure Injury Advisory Panel. Accessed October 9, 2022. https://npiap.com/page/2019Guideline
2. Ricci JA, Bayer LR, Orgill DP. Evidence-based medicine: the evaluation and treatment of pressure injuries. Plast Reconstr Surg. 2017;139:275e-286e. doi: 10.1097/PRS.0000000000002850
1. 2019 Guideline. The National Pressure Injury Advisory Panel. Accessed October 9, 2022. https://npiap.com/page/2019Guideline
2. Ricci JA, Bayer LR, Orgill DP. Evidence-based medicine: the evaluation and treatment of pressure injuries. Plast Reconstr Surg. 2017;139:275e-286e. doi: 10.1097/PRS.0000000000002850
Newborn with white oral lesions
These lesions, called Bohn nodules, manifest on the buccal or lingual portion of the maxillary alveolar ridge and, less frequently, on the mandibular alveolar ridge. Because Bohn nodules are white and have a firm consistency, they are often confused with teeth. They can be differentiated by location, as teeth usually erupt from the distal aspect of the alveolar ridge.
Bohn nodules are epithelial cysts that are filled with keratin, which gives them their white color. They are caused by portions of epithelium that get trapped under surrounding epithelial cells. Bohn nodules usually resolve when the overlying epithelium ruptures and releases the keratinaceous material (usually by the time the child is 3 months of age).1
These nodules can be confused with neonatal or supernumerary teeth. Neonatal teeth can be abnormally small and pointed; they are true deciduous teeth that have erupted early. If they are removed, the child will not replace them until the time of their adult tooth eruption. Additionally, there are supernumerary teeth, which are extra teeth that are often abnormally shaped and loosely adherent. These abnormal teeth warrant extraction to avoid trauma to the tongue or aspiration.2
In this case, the family was advised regarding the benign nature of the nodules and the expectation that they would spontaneously resolve.
Images and text courtesy of Daniel Stulberg, MD, FAAFP, Professor and Chair, Department of Family and Community Medicine, Western Michigan University Homer Stryker, MD School of Medicine Kalamazoo.
1. Gupta N, Ramji S. Bohn's nodules: an under-recognised entity. Arch Dis Child Fetal Neonatal Ed. 2013;98:F464. doi: 10.1136/archdischild-2012-302922
2. DeSeta M, Holden E, Siddik D, et al. Natal and neonatal teeth: a review and case series. Br Dent J. 2022;232:449-453. doi: 10.1038/s41415-022-4091-3
These lesions, called Bohn nodules, manifest on the buccal or lingual portion of the maxillary alveolar ridge and, less frequently, on the mandibular alveolar ridge. Because Bohn nodules are white and have a firm consistency, they are often confused with teeth. They can be differentiated by location, as teeth usually erupt from the distal aspect of the alveolar ridge.
Bohn nodules are epithelial cysts that are filled with keratin, which gives them their white color. They are caused by portions of epithelium that get trapped under surrounding epithelial cells. Bohn nodules usually resolve when the overlying epithelium ruptures and releases the keratinaceous material (usually by the time the child is 3 months of age).1
These nodules can be confused with neonatal or supernumerary teeth. Neonatal teeth can be abnormally small and pointed; they are true deciduous teeth that have erupted early. If they are removed, the child will not replace them until the time of their adult tooth eruption. Additionally, there are supernumerary teeth, which are extra teeth that are often abnormally shaped and loosely adherent. These abnormal teeth warrant extraction to avoid trauma to the tongue or aspiration.2
In this case, the family was advised regarding the benign nature of the nodules and the expectation that they would spontaneously resolve.
Images and text courtesy of Daniel Stulberg, MD, FAAFP, Professor and Chair, Department of Family and Community Medicine, Western Michigan University Homer Stryker, MD School of Medicine Kalamazoo.
These lesions, called Bohn nodules, manifest on the buccal or lingual portion of the maxillary alveolar ridge and, less frequently, on the mandibular alveolar ridge. Because Bohn nodules are white and have a firm consistency, they are often confused with teeth. They can be differentiated by location, as teeth usually erupt from the distal aspect of the alveolar ridge.
Bohn nodules are epithelial cysts that are filled with keratin, which gives them their white color. They are caused by portions of epithelium that get trapped under surrounding epithelial cells. Bohn nodules usually resolve when the overlying epithelium ruptures and releases the keratinaceous material (usually by the time the child is 3 months of age).1
These nodules can be confused with neonatal or supernumerary teeth. Neonatal teeth can be abnormally small and pointed; they are true deciduous teeth that have erupted early. If they are removed, the child will not replace them until the time of their adult tooth eruption. Additionally, there are supernumerary teeth, which are extra teeth that are often abnormally shaped and loosely adherent. These abnormal teeth warrant extraction to avoid trauma to the tongue or aspiration.2
In this case, the family was advised regarding the benign nature of the nodules and the expectation that they would spontaneously resolve.
Images and text courtesy of Daniel Stulberg, MD, FAAFP, Professor and Chair, Department of Family and Community Medicine, Western Michigan University Homer Stryker, MD School of Medicine Kalamazoo.
1. Gupta N, Ramji S. Bohn's nodules: an under-recognised entity. Arch Dis Child Fetal Neonatal Ed. 2013;98:F464. doi: 10.1136/archdischild-2012-302922
2. DeSeta M, Holden E, Siddik D, et al. Natal and neonatal teeth: a review and case series. Br Dent J. 2022;232:449-453. doi: 10.1038/s41415-022-4091-3
1. Gupta N, Ramji S. Bohn's nodules: an under-recognised entity. Arch Dis Child Fetal Neonatal Ed. 2013;98:F464. doi: 10.1136/archdischild-2012-302922
2. DeSeta M, Holden E, Siddik D, et al. Natal and neonatal teeth: a review and case series. Br Dent J. 2022;232:449-453. doi: 10.1038/s41415-022-4091-3
SCLC Guidelines
Chronic Kidney Disease in People with Type 2 Diabetes
We know from the literature and in practice that type 2 diabetes (T2D) is one of the most common risk factors for developing chronic kidney disease (CKD). How prevalent is this overlap, and are certain patients more at risk than others?
Dr. McGill: That’s correct, in fact, 20% to 40% of people with T2D have identifiable CKD, and the rest are at risk for developing CKD in the future. All patients with T2D should recognize that risk and undergo annual screening for CKD. If an individual has prediabetes, then step up the screening to perhaps twice a year to see if the person has progressed. At that point, we can think about intervening with a medication to prevent the onset of diabetes, particularly if the patient is unable to adopt significant lifestyle changes.
In your day-to-day practice, what therapeutic approach do you take in managing patients with T2D and CKD?
Dr. McGill: The earliest and arguably the most important treatment for the care of CKD in T2D is glucose control. Establishing and maintaining blood glucose levels near the normal range is our strongest weapon for preventing CKD. Another treatment avenue is controlling blood pressure. The American Diabetes Association and other groups recommend that blood pressure be less than 130/80 mm Hg. It is critical that we treat hypertension effectively to achieve those numbers.
We also have therapies, such as the SGLT2 inhibitors, that offer protection from progression of CKD and from hospitalization for heart failure. Deployment of these newer agents is important for people who have more advanced diabetes or other serious health conditions.
What is the rate of disease progression, related complications, or even mortality for these patients?
Dr. McGill: People with CKD and T2D are at risk for many things. One risk is progression of kidney disease all the way to end-stage kidney disease, which requires dialysis or transplantation. Another huge risk is cardiovascular events such as myocardial infarction (MI) and stroke.
Persons with kidney disease, for reasons we don't understand, are at higher risk of MI and stroke than people who do not have kidney disease. Therefore, the risks of early mortality and events that adversely affect quality of life are greatly increased.
Can you please discuss the economic burdens associated with T2D and CKD, and whether any interventions are in place to help offset those costs?
Dr. McGill: Diabetes itself is wickedly expensive. We have excellent treatments for diabetes today, but they are very costly. The best approach for the prevention of diabetes is to be screened. When a patient presents with prediabetes, it’s important that they take important measures, such as weight loss, exercising 150 minutes per week, or reducing 500 calories a day from their diet, all of which have been shown to forestall the onset of diabetes.
Once diabetes develops, achieving near-normal glucose control can either be very inexpensive with one or more generic drugs, or it can be terribly expensive with the newer branded drugs. Both options can help with the achievement of near-normal glucose levels, but the newer drugs are better for weight loss and provide protection from heart and kidney disease.
It is important to consider where the patient is along the disease spectrum, and to educate them on the benefits of taking a proactive approach to their health. Don’t wait for diabetes to develop before doing something about it. We have to take action earlier, and more definitively.
We do everything we can to help patients with the high cost of diabetes medications. Pharma companies offer various coupons and patient assistance programs, but it's really important that we get people on the right therapy. In order for that to happen, they have to come to office visits and get lab tests done.
Is there anything else you would like to share on this topic?
Dr. McGill: Once a person has been diagnosed with diabetes, then excellent glucose control from the onset has been shown to prevent later complications, and early treatment is inexpensive. As people progress through their journey with diabetes and blood sugars go up, we have excellent therapies that help manage high glucose and help with weight loss.
We have to be realistic and rethink our approach in some ways, but as long as people develop good health care habits and visit the doctor once or twice a year specifically to address diabetes and blood pressure, we might be able to avoid long-term complications.
We know from the literature and in practice that type 2 diabetes (T2D) is one of the most common risk factors for developing chronic kidney disease (CKD). How prevalent is this overlap, and are certain patients more at risk than others?
Dr. McGill: That’s correct, in fact, 20% to 40% of people with T2D have identifiable CKD, and the rest are at risk for developing CKD in the future. All patients with T2D should recognize that risk and undergo annual screening for CKD. If an individual has prediabetes, then step up the screening to perhaps twice a year to see if the person has progressed. At that point, we can think about intervening with a medication to prevent the onset of diabetes, particularly if the patient is unable to adopt significant lifestyle changes.
In your day-to-day practice, what therapeutic approach do you take in managing patients with T2D and CKD?
Dr. McGill: The earliest and arguably the most important treatment for the care of CKD in T2D is glucose control. Establishing and maintaining blood glucose levels near the normal range is our strongest weapon for preventing CKD. Another treatment avenue is controlling blood pressure. The American Diabetes Association and other groups recommend that blood pressure be less than 130/80 mm Hg. It is critical that we treat hypertension effectively to achieve those numbers.
We also have therapies, such as the SGLT2 inhibitors, that offer protection from progression of CKD and from hospitalization for heart failure. Deployment of these newer agents is important for people who have more advanced diabetes or other serious health conditions.
What is the rate of disease progression, related complications, or even mortality for these patients?
Dr. McGill: People with CKD and T2D are at risk for many things. One risk is progression of kidney disease all the way to end-stage kidney disease, which requires dialysis or transplantation. Another huge risk is cardiovascular events such as myocardial infarction (MI) and stroke.
Persons with kidney disease, for reasons we don't understand, are at higher risk of MI and stroke than people who do not have kidney disease. Therefore, the risks of early mortality and events that adversely affect quality of life are greatly increased.
Can you please discuss the economic burdens associated with T2D and CKD, and whether any interventions are in place to help offset those costs?
Dr. McGill: Diabetes itself is wickedly expensive. We have excellent treatments for diabetes today, but they are very costly. The best approach for the prevention of diabetes is to be screened. When a patient presents with prediabetes, it’s important that they take important measures, such as weight loss, exercising 150 minutes per week, or reducing 500 calories a day from their diet, all of which have been shown to forestall the onset of diabetes.
Once diabetes develops, achieving near-normal glucose control can either be very inexpensive with one or more generic drugs, or it can be terribly expensive with the newer branded drugs. Both options can help with the achievement of near-normal glucose levels, but the newer drugs are better for weight loss and provide protection from heart and kidney disease.
It is important to consider where the patient is along the disease spectrum, and to educate them on the benefits of taking a proactive approach to their health. Don’t wait for diabetes to develop before doing something about it. We have to take action earlier, and more definitively.
We do everything we can to help patients with the high cost of diabetes medications. Pharma companies offer various coupons and patient assistance programs, but it's really important that we get people on the right therapy. In order for that to happen, they have to come to office visits and get lab tests done.
Is there anything else you would like to share on this topic?
Dr. McGill: Once a person has been diagnosed with diabetes, then excellent glucose control from the onset has been shown to prevent later complications, and early treatment is inexpensive. As people progress through their journey with diabetes and blood sugars go up, we have excellent therapies that help manage high glucose and help with weight loss.
We have to be realistic and rethink our approach in some ways, but as long as people develop good health care habits and visit the doctor once or twice a year specifically to address diabetes and blood pressure, we might be able to avoid long-term complications.
We know from the literature and in practice that type 2 diabetes (T2D) is one of the most common risk factors for developing chronic kidney disease (CKD). How prevalent is this overlap, and are certain patients more at risk than others?
Dr. McGill: That’s correct, in fact, 20% to 40% of people with T2D have identifiable CKD, and the rest are at risk for developing CKD in the future. All patients with T2D should recognize that risk and undergo annual screening for CKD. If an individual has prediabetes, then step up the screening to perhaps twice a year to see if the person has progressed. At that point, we can think about intervening with a medication to prevent the onset of diabetes, particularly if the patient is unable to adopt significant lifestyle changes.
In your day-to-day practice, what therapeutic approach do you take in managing patients with T2D and CKD?
Dr. McGill: The earliest and arguably the most important treatment for the care of CKD in T2D is glucose control. Establishing and maintaining blood glucose levels near the normal range is our strongest weapon for preventing CKD. Another treatment avenue is controlling blood pressure. The American Diabetes Association and other groups recommend that blood pressure be less than 130/80 mm Hg. It is critical that we treat hypertension effectively to achieve those numbers.
We also have therapies, such as the SGLT2 inhibitors, that offer protection from progression of CKD and from hospitalization for heart failure. Deployment of these newer agents is important for people who have more advanced diabetes or other serious health conditions.
What is the rate of disease progression, related complications, or even mortality for these patients?
Dr. McGill: People with CKD and T2D are at risk for many things. One risk is progression of kidney disease all the way to end-stage kidney disease, which requires dialysis or transplantation. Another huge risk is cardiovascular events such as myocardial infarction (MI) and stroke.
Persons with kidney disease, for reasons we don't understand, are at higher risk of MI and stroke than people who do not have kidney disease. Therefore, the risks of early mortality and events that adversely affect quality of life are greatly increased.
Can you please discuss the economic burdens associated with T2D and CKD, and whether any interventions are in place to help offset those costs?
Dr. McGill: Diabetes itself is wickedly expensive. We have excellent treatments for diabetes today, but they are very costly. The best approach for the prevention of diabetes is to be screened. When a patient presents with prediabetes, it’s important that they take important measures, such as weight loss, exercising 150 minutes per week, or reducing 500 calories a day from their diet, all of which have been shown to forestall the onset of diabetes.
Once diabetes develops, achieving near-normal glucose control can either be very inexpensive with one or more generic drugs, or it can be terribly expensive with the newer branded drugs. Both options can help with the achievement of near-normal glucose levels, but the newer drugs are better for weight loss and provide protection from heart and kidney disease.
It is important to consider where the patient is along the disease spectrum, and to educate them on the benefits of taking a proactive approach to their health. Don’t wait for diabetes to develop before doing something about it. We have to take action earlier, and more definitively.
We do everything we can to help patients with the high cost of diabetes medications. Pharma companies offer various coupons and patient assistance programs, but it's really important that we get people on the right therapy. In order for that to happen, they have to come to office visits and get lab tests done.
Is there anything else you would like to share on this topic?
Dr. McGill: Once a person has been diagnosed with diabetes, then excellent glucose control from the onset has been shown to prevent later complications, and early treatment is inexpensive. As people progress through their journey with diabetes and blood sugars go up, we have excellent therapies that help manage high glucose and help with weight loss.
We have to be realistic and rethink our approach in some ways, but as long as people develop good health care habits and visit the doctor once or twice a year specifically to address diabetes and blood pressure, we might be able to avoid long-term complications.
Psychiatrist sentenced to 11 years for sledgehammer attack against another psychiatrist
of her child’s father, who is also a psychiatrist.
Pamela Buchbinder pled guilty to first-degree burglary and assault on September 7, almost exactly 10 years after the November 2012 attack on Michael Weiss, MD. Weiss was beaten with a sledgehammer and stabbed multiple times but survived the attack.
The September plea deal was announced by the Manhattan district attorney, who said that Ms. Buchbinder acknowledged she had enlisted the help of her then-19-year-old cousin Jacob Nolan to kill Dr. Weiss. Ms. Buchbinder was in a custody battle with Dr. Weiss over their then-5-year-old child.
At the Oct. 11 sentencing, Ms. Buchbinder and her defense attorney attempted to withdraw that plea. NBC4 New York reported that Buchbinder claimed she was not in her right mind on the day of the plea because she had received a “contact high” from others in her holding cell who were using synthetic marijuana and that she had not taken her prescribed medications that day.
The judge did not entertain the request and proceeded with the sentencing.
Ms. Buchbinder has been held at Rikers Island prison, in East Elmhurst, N.Y., since she was arrested in 2017, so has already served 5 years of her 11-year sentence. She must also serve 5 years of postrelease probation.
Insurance policy beneficiary
Ms. Buchbinder’s cousin was convicted of second-degree attempted murder in 2016 and was sentenced to 9.5 years in prison.
In a 2017 interview with CBS News, Mr. Nolan, who said he was “bipolar,” claimed Ms. Buchbinder had manipulated him into trying to kill her child’s father by telling him “horror stories” about Weiss. Soon after the interview, Ms. Buchbinder was arrested.
In 2022, the New York Post reported that Ms. Buchbinder had been named a beneficiary of Dr. Weiss’ $1.5 million life insurance policy several days before the attack.
Prosecutors had surveillance footage of Ms. Buchbinder with Nolan at a Manhattan hardware store purchasing the sledgehammer. According to the CBS report, at the time of her arrest, she also was apparently preparing to flee.
She was denied bail and has been held at Rikers Island since her arrest.
Ms. Buchbinder was licensed to practice in New York in 1999. In April 2018, the New York State Board for Professional Medical Conduct issued an interim order that precluded her from practicing medicine in New York.
The interim order will be in effect until the board completes its investigation. As of press time, the board had not updated its files.
A version of this article first appeared on Medscape.com.
of her child’s father, who is also a psychiatrist.
Pamela Buchbinder pled guilty to first-degree burglary and assault on September 7, almost exactly 10 years after the November 2012 attack on Michael Weiss, MD. Weiss was beaten with a sledgehammer and stabbed multiple times but survived the attack.
The September plea deal was announced by the Manhattan district attorney, who said that Ms. Buchbinder acknowledged she had enlisted the help of her then-19-year-old cousin Jacob Nolan to kill Dr. Weiss. Ms. Buchbinder was in a custody battle with Dr. Weiss over their then-5-year-old child.
At the Oct. 11 sentencing, Ms. Buchbinder and her defense attorney attempted to withdraw that plea. NBC4 New York reported that Buchbinder claimed she was not in her right mind on the day of the plea because she had received a “contact high” from others in her holding cell who were using synthetic marijuana and that she had not taken her prescribed medications that day.
The judge did not entertain the request and proceeded with the sentencing.
Ms. Buchbinder has been held at Rikers Island prison, in East Elmhurst, N.Y., since she was arrested in 2017, so has already served 5 years of her 11-year sentence. She must also serve 5 years of postrelease probation.
Insurance policy beneficiary
Ms. Buchbinder’s cousin was convicted of second-degree attempted murder in 2016 and was sentenced to 9.5 years in prison.
In a 2017 interview with CBS News, Mr. Nolan, who said he was “bipolar,” claimed Ms. Buchbinder had manipulated him into trying to kill her child’s father by telling him “horror stories” about Weiss. Soon after the interview, Ms. Buchbinder was arrested.
In 2022, the New York Post reported that Ms. Buchbinder had been named a beneficiary of Dr. Weiss’ $1.5 million life insurance policy several days before the attack.
Prosecutors had surveillance footage of Ms. Buchbinder with Nolan at a Manhattan hardware store purchasing the sledgehammer. According to the CBS report, at the time of her arrest, she also was apparently preparing to flee.
She was denied bail and has been held at Rikers Island since her arrest.
Ms. Buchbinder was licensed to practice in New York in 1999. In April 2018, the New York State Board for Professional Medical Conduct issued an interim order that precluded her from practicing medicine in New York.
The interim order will be in effect until the board completes its investigation. As of press time, the board had not updated its files.
A version of this article first appeared on Medscape.com.
of her child’s father, who is also a psychiatrist.
Pamela Buchbinder pled guilty to first-degree burglary and assault on September 7, almost exactly 10 years after the November 2012 attack on Michael Weiss, MD. Weiss was beaten with a sledgehammer and stabbed multiple times but survived the attack.
The September plea deal was announced by the Manhattan district attorney, who said that Ms. Buchbinder acknowledged she had enlisted the help of her then-19-year-old cousin Jacob Nolan to kill Dr. Weiss. Ms. Buchbinder was in a custody battle with Dr. Weiss over their then-5-year-old child.
At the Oct. 11 sentencing, Ms. Buchbinder and her defense attorney attempted to withdraw that plea. NBC4 New York reported that Buchbinder claimed she was not in her right mind on the day of the plea because she had received a “contact high” from others in her holding cell who were using synthetic marijuana and that she had not taken her prescribed medications that day.
The judge did not entertain the request and proceeded with the sentencing.
Ms. Buchbinder has been held at Rikers Island prison, in East Elmhurst, N.Y., since she was arrested in 2017, so has already served 5 years of her 11-year sentence. She must also serve 5 years of postrelease probation.
Insurance policy beneficiary
Ms. Buchbinder’s cousin was convicted of second-degree attempted murder in 2016 and was sentenced to 9.5 years in prison.
In a 2017 interview with CBS News, Mr. Nolan, who said he was “bipolar,” claimed Ms. Buchbinder had manipulated him into trying to kill her child’s father by telling him “horror stories” about Weiss. Soon after the interview, Ms. Buchbinder was arrested.
In 2022, the New York Post reported that Ms. Buchbinder had been named a beneficiary of Dr. Weiss’ $1.5 million life insurance policy several days before the attack.
Prosecutors had surveillance footage of Ms. Buchbinder with Nolan at a Manhattan hardware store purchasing the sledgehammer. According to the CBS report, at the time of her arrest, she also was apparently preparing to flee.
She was denied bail and has been held at Rikers Island since her arrest.
Ms. Buchbinder was licensed to practice in New York in 1999. In April 2018, the New York State Board for Professional Medical Conduct issued an interim order that precluded her from practicing medicine in New York.
The interim order will be in effect until the board completes its investigation. As of press time, the board had not updated its files.
A version of this article first appeared on Medscape.com.
PsA Guidelines
Randomized, Double-Blind Placebo-Controlled Trial to Assess the Effect of Probiotics on Irritable Bowel Syndrome in Veterans With Gulf War Illness
About 700,000 US military personnel were deployed in Operation Desert Storm (August 1990 to March 1991).1 Almost 30 years since the war, a large number of these veterans continue to experience a complex of symptoms of unknown etiology called Gulf War illness (GWI), which significantly affects health and quality of life (QOL). The lack of clear etiology of the illness has impaired research to find specific treatments and has further exacerbated the stress among veterans. GWI typically includes a mixture of chronic headache, cognitive difficulties, widespread pain, unexplained fatigue, memory and concentration problems, as well as chronic respiratory and gastrointestinal (GI) symptoms.2 Abdominal pain and alteration of bowel habits are also symptoms typical of irritable bowel syndrome (IBS). It has been estimated that IBS occurs in up to 30% of Gulf War veterans.3
The etiology of IBS is unknown. Possible mechanisms include visceral hypersensitivity, altered gut motor function, aberrant brain-gut interaction, and psychological factors, perhaps with a genetic predisposition.4 Gastroenteritis has been reported as a triggering mechanism in up to one-third of patients with IBS.5 Gastroenteritis can alter the gut microbiota and has been reported to be a significant risk factor for the development of IBS.6 In one study of Operation Desert Shield soldiers, > 50% of military personnel developed acute gastroenteritis while on duty.7 A high prevalence of extra-intestinal symptoms also has been reported, including fatigue, headache, joint pains, and anxiety, in Gulf War veterans with IBS. These extra-intestinal symptoms of IBS are consistent with the reported GWI symptoms. Change in gut microbiota also has been associated with many of the extra-intestinal symptoms of IBS, especially fatigue.8,9 Gut microbiota are known to change with travel, stress, and a change in diet, all potential factors that are relevant to Gulf War veterans. This would suggest that an imbalance in the gut microbiota, ie, dysbiosis, may play a role in the pathogenesis of both IBS and GWI. Dysbiosis could be a risk factor for or alternatively a consequence of GWI.
A systematic review highlighted the heterogeneity of the gut microbiota in patients with IBS.10 Overall, Enterobacteriaceae, Lactobacillaceae, and Bacteroides were increased, whereas Clostridiales, Faecalibacterium, and Bifidobacterium were decreased in patients with IBS compared with controls. Gut microbiota also has been associated with cognitive changes, anxiety, and depression—symptoms associated with IBS and are part of the GWI.
If altered gut microbiota contributes to the etiopathogenesis of IBS, its restoration of with probiotics should help. Probiotics are live organisms that when ingested may improve health by promoting the growth of naturally occurring flora and establishing a healthy gut flora. Probiotics have several mechanisms of actions. Probiotics work in the lumen of the gut by producing antibacterial molecules and enhancing the mucosal barrier.11 Probiotics also may produce metabolic compounds that alter the intestinal microbiota and improve intestinal barrier function.12 Probiotics also have been shown to activate receptors in the enteric nervous system with the potential to promote pain relief in the setting of visceral hyperalgesia.13,14 The anti-inflammatory properties of probiotics potentially could modulate the basic pathophysiology of IBS and improve motility, visceral hypersensitivity, and brain-gut interaction.15 Furthermore, significant gut dysbiosis has been shown with GWI; suggesting that probiotics may have a role in its management.16,17
Probiotics have not been studied in Gulf War veterans with IBS. We performed a prospective, double-blind placebo-controlled study to determine the efficacy of a commercially available probiotic containing 8 strains of bacteria (De Simone Formulation; formally known as VSL#3 and Visbiome) on symptoms of IBS and GWI. This probiotic was selected as the overall literature suggested benefit of combination probiotics in IBS, and VSL#3 has been shown to be efficacious in ulcerative colitis and microscopic colitis.18-20
Methods
Veterans who served in Operation Desert Storm (August 1990 to March 1991) and enrolled at the George E. Wahlen Veterans Affairs (VA) Medical Center (GEWVAMC), Salt Lake City, Utah, were eligible for the study. The inclusion criteria were: veterans aged ≥ 35 years; ≥ 2 nonintestinal GWI symptoms (eg, fatigue, joint pains, insomnia, general stiffness, and headache); IBS diagnosis based on the Rome III criteria; IBS symptoms > 6 months; normal gross appearance of the colonic mucosa; negative markers for celiac disease and inflammatory bowel disease (IBD); normal thyroid function; and serum calcium levels.21 Those who had a clinically significant cardiac, pulmonary, hepatic or renal dysfunction; history of/or presence of systemic malignancy; current evidence of celiac disease or IBD; unstable/significant psychiatric disease; recent change in GI medications; current pregnancy; or use of antibiotics or probiotics within the past 1 month were excluded. Subjects were enrolled from a list of veterans with GWI from the GEWVAMC Gulf War registry; referrals to gastroenterology clinics for IBS from internal medicine clinics; and posted advertisements.
Protocol
After written informed consent was obtained, each veteran was verified to have IBS and ≥ 2 GWI symptoms. All veterans had the following tests and panels: complete blood count, erythrocyte sedimentation rate, serum comprehensive metabolic panel, thyroid-stimulating hormone, tissue transglutaminase, stool test for ova and parasite, giardia antigen, and clostridia toxins to exclude organic cause of GI symptoms. Colonoscopy was performed in all veterans to exclude IBD, and to rule out microscopic or lymphocytic colitis.
Randomization was computer generated and maintained by the study pharmacist so that study personnel and patients were blinded to the trial groups. All investigators were blinded and allocation was concealed. The medication was supplied in a numbered container by the pharmacist after patient enrollment. After a 2-week run-in period, veterans were randomized (1:1) to receive either 1 sachet of probiotic (De Simone Formulation; formally known as VSL#3 and Visbiome) or placebo once daily for 8 weeks.
Each probiotic packet contains 900 billion probiotic bacteria per sachet.11 This formulation contained 8 viable strains of bacteria: 4 strains of Lactobacillus (L acidophilus, L plantarum, L paracasei, L delbrueckii subsp. bulgaricus); 3 strains of Bifidobacteria (Bifidobacterium breve, B lactis, B infantis); and 1 strain of Streptococcus thermophilus. This formulation had been commercialized and studied as VSL#3 and is currently available in the United States under the Visbiome trade name. While branding changed during the study, the formulation did not. The investigational medicine (VSL#3, Visbiome, and placebo) were shipped from the manufacturer Dupont/Danisco in Madison, Wisconsin. The subjects received placebo or probiotic (VSL#3/Visbiome) and both were identical in appearance. The medication was supplied in a numbered container by the pharmacist after patient enrollment.
Measures
Veterans completed the bowel disease questionnaire to record baseline bowel habits.22 All veterans recorded daily bowel symptoms to confirm the presence of IBS during the 2-week pretreatment period, at baseline, and at the end of the 8-week treatment. The symptoms assessed included severity of abdominal pain (0, none to 100, severe); severity of bloating (0, none to 100, severe); stool frequency; Bristol stool scale (1, very hard to 7, watery); severity of diarrhea (0, none to 100, severe); severity of constipation (0, none to 100, severe); satisfaction with bowel habits (0, none to 100, severe); and IBS affecting or interfering with life (0, none to 100, severe). The bowel symptom score is the sum of the 5 symptom scores.23,24
IBS-specific QOL (IBS-QOL) was recorded at baseline and at the end of treatment.25 The IBS-QOL consists of a 34-item validated disease-specific questionnaire that measures 8 domains relevant to subjects with IBS: dysphoria, interference with activity, body image, health worry, food avoidance, social reaction, sexual life, and relationships. We used the Somatic Symptom Checklist to detect the following extra-intestinal symptoms that are common among veterans with GWI: headache, backache, wheeziness, insomnia, bad breath, fatigue, general stiffness, dizziness, weakness, sensitivity to hot and cold, palpitation, and tightness in chest. Subjects rated symptoms on a scale of 1 to 5: how often (1, none; 2, monthly; 3, once weekly; 4, several times weekly; 5, daily), and how bothersome (1, not at all to 5, extremely).26
Subjects completed the Posttraumatic Stress Disorder (PTSD) Checklist–Military, which is specific to military experience with 17 items on a 1 to 5 scale (1, not at all to 5, extremely). Scores were summed to produce a total symptom severity score (range, 17-85).27 Subjects also completed the Brief Symptom Inventory 18 (BSI-18) during the baseline evaluation.28 BSI-18 measures subjects’ reported overall psychological distress. It assesses 3 symptoms dimensions (somatization, depression, and anxiety) and a global severity index. The raw scores were transferred to normative T scores based on samples of nonpatient normal men and women.
Symptom data were compared after 8 weeks of treatment. The primary study endpoint was change in bowel symptom score. The secondary endpoints were mean change in symptoms, QOL, extra-intestinal symptoms, and PTSD score. The study was approved by the Salt Lake City Veterans Affairs Medical Center and the University of Utah Institutional Review Board and registered in ClinicalTrials.gov (NCT03078530).
Statistical Methods
Comparisons of the probiotic vs placebo groups for demographic variable were analyzed using a 2-sample t test for continuous variables, and with a χ2 test or Fisher exact test for categorical variables. The primary and secondary outcome variables were recorded daily for 2 weeks as pretreatment baseline and for 2 weeks at the end of treatment. These symptoms were recorded as ordered categorical variables, which were then averaged across the week to produce a continuous measurement for statistical analysis. For the primary outcome of GI symptoms, posttreatment comparisons were made between the study groups using a 2-sample t test of the baseline vs posttreatment values. All P values were calculated for 2-sided comparisons. The planned sample size in our study protocol was to recruit 40 individuals per group in order to achieve 80% power to detect a 30% improvement between baseline and end of treatment in the primary bowel symptom score. This study recruited 53 subjects. With this sample size, the study had 80% power to detect a 0.8 SD in any of the outcomes.
Results
We screened 101 veterans with IBS and GWI; 39 veterans did not fulfill the inclusion/exclusion criteria, 22 declined to participate or did not complete the screening questionnaires and tests, and 9 were lost to follow-up. Sixty-two participants were randomized in a double-blind placebo-controlled study design; 9 dropped out before the end of the study. Data were analyzed from 53 veterans who completed the study, 29 in the placebo group and 24 in the probiotic group (Figure 1). The cohort was primarily male with a mean (SD) age of 55 (8) years (range, 42-73) (Table 1).
Overall, the treatment was well tolerated. All subjects were contacted every 2 weeks during the study to check for adverse effects, but no serious events were reported. There were no differences at baseline in any of the BSI-18 subscale scores in veterans between the groups. There was a greater mean (SEM) improvement of diarrhea severity in the probiotic group compared with the placebo group: 18 (6), a 31% improvement, vs 6 (5), a 13% improvement, respectively; however, the difference was not statistically significance (P = .13) (Table 2). There also was a greater mean (SEM) improvement in satisfaction of bowel habits in the probiotic group compared with the placebo group: 16 (7), a 35% improvement vs 4 (9), an 8% worsening; this also was not statistically significant (P = .09). There was no difference in the change of IBS-QOL before and after treatment in either group (Figure 2). There was no improvement in any of the symptoms of GWI (all P ≥ .06) (Appendix).
Discussion
GWI is a complex multisystem illness of unknown etiology. There was high prevalence of diarrhea during deployment, and veterans were exposed to several physical, environmental, and mental stresses of the war.3 A change in gut microbiota can occur during deployment due to diet changes, environmental and physical stress, and GI infections.29 These changes would suggest that manipulation of gut microbiota might offer a new modality of treatment of IBS and GWI. We evaluated the effect of a high-potency multistrain probiotic in veterans with IBS and GWI. We did not detect any statistically significant differences between the probiotic and placebo groups on bowel symptom score and individual symptoms of IBS and on QOL. Also, there was no improvement for the other symptoms of GWI. To our knowledge, this is the first study evaluating the effect of probiotics in veterans with IBS and GWI. Our results are consistent with the literature on probiotics and IBS.
The probiotic formulation used in our study has been evaluated in patients with IBS previously. Kim and colleagues found that after 8 weeks of treatment of patients with diarrhea-predominant IBS with VSL#3, there was improvement in bloating, but no effect was found on abdominal pain, gas, or urgency.30 A subsequent study by the same investigators on patients with all types of IBS found that VSL#3 showed no effect on abdominal pain, stool frequency and consistency, or on bloating, but there was improvement in flatulence.31 Another study that evaluated the effect of VSL#3 on symptoms of diarrhea-predominant IBS and QOL found improvement in IBS symptoms from baseline in both the probiotic and the placebo groups, but the difference between the 2 groups was not statistically significant.32 Similarly, Wong and colleagues performed a double-blind, placebo-controlled mechanistic study to evaluate the effect of VSL#3. They found improvement in bowel symptom score, abdominal pain intensity, and satisfaction with bowel habits with both the VSL#3 and placebo group but similar to our study, the differences were not statistically significant.
Several reviews have evaluated the efficacy of probiotics for IBS. A 2010 review found evidence that probiotics trended toward improved IBS symptoms compared with placebo.33 The 2014 follow-up by the same authors demonstrated that overall, probiotics improved global symptoms of IBS and multistrain probiotics were more effective.20 A third meta-analysis from the same group found evidence that multistrain probiotics seemed to have a beneficial effect but could not definitively conclude that probiotics are efficacious in improving IBS symptoms.34 Other authors also have seen inconsistent effects of probiotics compared with placebo on global symptoms, abdominal pain, and bloating after performing systematic reviews of the literature.35-38 Although several reviews support that multistrain probiotics are more effective, they fail to conclude which combinations are more efficacious.
The effect of probiotics on QOL has not been investigated by many studies.37 In our study, we did not find significant improvement in QOL in the probiotic group, which is in line with 2 previous studies that showed no effect on IBS QOL of VSL#3 vs placebo.32,39 Most of the research reports that multistrain probiotics are more effective than using a single strain.34,35,40Bifidobacterium and Lactobacillus are the most commonly used bacteria in the multistrain probiotics that have shown their positive effect on IBS.35,41 The probiotic used in our study contained other species along with these 2 microorganisms.
The dose and duration of treatment of probiotics also has been debated. In one meta-analysis, the investigators found that studies of ≥ 8 weeks were more likely to show a positive effect; 4 of the 7 studies with statistically significant improvement in IBS symptoms were longer than 8 weeks.35 However, another meta-analysis based on 35 randomized controlled trials found that there was not a statistically significant difference between groups treated for > 4 weeks vs < 4 weeks.42 In addition, another meta-analysis of VSL#3 on IBS in children and adults also found no difference in results based on the duration of treatment of probiotics.43 Similar to our study, 3 other studies of VSL#3 treated patients for 8 weeks and found no statistically significant effect.30-32 In the past, VSL#3 has been used at dosages of 450 or 900 billion bacteria per day.
An individual’s response to probiotics may depend on the subtype of IBS. However, most of the studies, like ours, included groups of all subtypes. It may be that probiotics are more effective in patients with moderate-to-severe symptoms. Most of our patients had milder symptoms, and we cannot discount how subjects with more severe disease may have responded to the drug. Interestingly, one study demonstrated that Lactobacillus was more effective in patients with moderately severe abdominal pain compared with mild symptoms.44
In our study, the probiotic did not improve PTSD symptoms or other extra-intestinal symptoms common in IBS and GWI. Similar to our study, Wong and colleagues did not find significant improvement of psychological and sleep scores after treatment with VSL#3.6 Similarly, there is evidence that alteration in gut microbiota is associated with health and diseases, but what specific alterations occur and whether they can be improved with probiotics remains unknown.45
Limitations
The inconsistent response to probiotics in various studies may be due to IBS heterogeneity. Furthermore, there are demographic differences between Gulf War veterans and patients enrolled in other studies: Gulf War veterans are predominantly male, many were deployed abroad and had a history of gastroenteritis during deployment, and were exposed to stressful situations.46 These factors may be involved in triggering or maintaining IBS in Gulf War veterans. A further limitation of our randomized trial is the relatively small sample size.
Conclusions
This study did not demonstrate statistically significant improvement in symptoms of IBS or improvement in QOL after treatment with a multistrain probiotic. We also did not find any improvement in symptoms of GWI or PTSD. There was no difference in psychological scores between the placebo and treatment groups, and it is unlikely that psychological factors confounded the response to treatment in this study.
The effectiveness of a probiotic may depend on the baseline gut microbiome of the individual and depend on the strain, amount, and frequency of bacteria used. A lack of response of the probiotics does not exclude gut viruses and fungi having a role in exacerbating GWI symptoms. It is also possible that the bacteria present or the dose of the probiotic used was not sufficient to improve symptoms. So far, the definitive benefit of probiotics has been demonstrated for only a few preparations, and none are approved by the US Food and Drug Administration for any disease. More research is needed to determine whether probiotics have any role in the treatment of IBS and GWI.
Acknowledgments
AKT received grant support from the US Department of Veterans Affairs and the US Department of Defense (W81XWH-10-1-0593, W81XWH-15-1-0636). We thank Keith G. Tolman, MD, for assistance in editing the initial proposal and for periodic consultation. We thank the manufacturer of the probiotic for supplying the active drug and the placebo. The manufacture of the probiotic had no role in the design and conduct of the study, analysis and interpretation of the data, and in the preparation of the manuscript.
1. O’Shea EF, Cotter PD, Stanton C, Ross RP, Hill C. Production of bioactive substances by intestinal bacteria as a basis for explaining probiotic mechanisms: bacteriocins and conjugated linoleic acid. Int J Food Microbiol. 2012;152(3):189-205. doi:10.1016/j.ijfoodmicro.2011.05.025.
2. Kamiya T, Wang L, Forsythe P, et al. Inhibitory effects of Lactobacillus reuteri on visceral pain induced by colorectal distension in Sprague-Dawley rats. Gut. 2006;55(2):191-196. doi:10.1136/gut.2005.070987.
3. Verdu EF, Bercik P, Verma-Gandhu M, et al. Specific probiotic therapy attenuates antibiotic induced visceral hypersensitivity in mice. Gut. 2006;55(2):182-190. doi:10.1136/gut.2005.066100
4. Ford AC, Harris LA, Lacy BE, Quigley EMM, Moayyedi P. Systematic review with meta-analysis: the efficacy of prebiotics, probiotics, synbiotics and antibiotics in irritable bowel syndrome. Aliment Pharmacol Ther. 2018;48(10):1044-1060. doi:10.1111/apt.15001.
5. Niu HL, Xiao JY. The efficacy and safety of probiotics in patients with irritable bowel syndrome: Evidence based on 35 randomized controlled trials. Int J Surg. 2020;75:116-127. doi:10.1016/j.ijsu.2020.01.142.
6. Wong RK, Yang C, Song GH, Wong J, Ho KY. Melatonin regulation as a possible mechanism for probiotic (VSL#3) in irritable bowel syndrome: a randomized double-blinded placebo study. Dig Dis Sci. 2015;60(1):186-194. doi:10.1007/s10620-014-3299-8.
7. Hyams KC, Bourgeois AL, Merrell BR, et al. Diarrheal disease during Operation Desert Shield. N Engl J Med. 1991;325(20):1423-1428. doi:10.1056/NEJM199111143252006 8. Clancy RL, Gleeson M, Cox A, et al. Reversal in fatigued athletes of a defect in interferon gamma secretion after administration of Lactobacillus acidophilus. Br J Sports Med. 2006;40(4):351-354. doi:10.1136/bjsm.2005.024364
9. Sullivan A, Nord CE, Evengard B. Effect of supplement with lactic-acid producing bacteria on fatigue and physical activity in patients with chronic fatigue syndrome. Nutr J. 2009;8:4. doi:10.1186/1475-2891-8-4
10. Pittayanon R, Lau JT, Yuan Y, et al. Gut microbiota in patients with irritable bowel syndrome—a systematic review. Gastroenterology. 2019;157(1):97-108. doi:10.1053/j.gastro.2019.03.049
11. Rao RK, Samak G. Protection and restitution of gut barrier by probiotics: nutritional and clinical implications. Curr Nutr Food Sci. 2013;9(2):99-107. doi:10.2174/1573401311309020004
12. O´Shea EF, Cotter PD, Stanton C, Ross RP, Hill C. Production of bioactive substances by intestinal bacteria as a basis for explaining probiotic mechanisms: bacteriocins and conjugated linoleic acid. Int J Food Microbiol. 2012;152(3):189-205. doi:10.1016/j.ijfoodmicro.2011.05.025
13. Kamiya T, Wang L, Forsythe P, et al. Inhibitory effects of Lactobacillus reuteri on visceral pain induced by colorectal distension in Sprague-Dawley rats. Gut. 2006;55(2):191-196. doi:10.1136/gut.2005.070987
14. Verdu EF, Bercik P, Verma-Gandhu M, et al. Specific probiotic therapy attenuates antibiotic induced visceral hypersensitivity in mice. Gut. 2006;55(2):182-190. doi:10.1136/gut.2005.06610015. O´Mahony L, McCarthy J, Kelly P, et al. Lactobacillus and bifidobacterium in irritable bowel syndrome: symptom responses and relationship to cytokine profiles. Gastroenterology. 2005;128(3):541-551. doi:10.1053/j.gastro.2004.11.050
16. Alhasson F, Das S, Seth R, et al. Altered gut microbiome in a mouse model of Gulf War Illness causes neuroinflammation and intestinal injury via leaky gut and TLR4 activation. PLoS One. 2017;12(3):e0172914. doi:10.1371/journal.pone.0172914.17. Janulewicz PA, Seth RK, Carlson JM, et al. The gut-microbiome in Gulf War veterans: a preliminary report. Int J Environ Res Public Health. 2019;16(19). doi:10.3390/ijerph16193751
18. Dang X, Xu M, Liu D, Zhou D, Yang W. Assessing the efficacy and safety of fecal microbiota transplantation and probiotic VSL#3 for active ulcerative colitis: a systematic review and meta-analysis. PLoS One. 2020;15(3):e0228846. doi:10.1371/journal.pone.0228846
19. Ford AC, Quigley EM, Lacy BE, et al. Efficacy of prebiotics, probiotics, and synbiotics in irritable bowel syndrome and chronic idiopathic constipation: systematic review and meta-analysis. Am J Gastroenterol. 2014;109(10):1547-1561; quiz 1546, 1562. doi:10.1038/ajg.2014.202
20. Rohatgi S, Ahuja V, Makharia GK, et al. VSL#3 induces and maintains short-term clinical response in patients with active microscopic colitis: a two-phase randomised clinical trial. BMJ Open Gastroenterol. 2015;2(1):e000018. doi:10.1136/bmjgast-2014-000018
21. Longstreth GF, Thompson WG, Chey WD, Houghton LA, Mearin F, Spiller RC. Functional bowel disorders. Gastroenterology. 2006;130(5):1480-1491. doi:10.1053/j.gastro.2005.11.061
22. Talley NJ, Phillips SF, Melton J, 3rd, Wiltgen C, Zinsmeister AR. A patient questionnaire to identify bowel disease. Ann Intern Med. 1989;111(8):671-674. doi:10.7326/0003-4819-111-8-671
23. Bensoussan A, Talley NJ, Hing M, Menzies R, Guo A, Ngu M. Treatment of irritable bowel syndrome with Chinese herbal medicine: a randomized controlled trial. JAMA. 1998;280(18):1585-1589. doi:10.1001/jama.280.18.1585
24. Francis CY, Morris J, Whorwell PJ. The irritable bowel severity scoring system: a simple method of monitoring irritable bowel syndrome and its progress. Aliment Pharmacol Ther. 1997;11(2):395-402. doi:10.1046/j.1365-2036.1997.142318000.x
25. Patrick DL, Drossman DA, Frederick IO, DiCesare J, Puder KL. Quality of life in persons with irritable bowel syndrome: development and validation of a new measure. Dig Dis Sci. 1998;43(2):400-411. doi:10.1023/a:1018831127942
26. Attanasio V, Andrasik F, Blanchard EB, Arena JG. Psychometric properties of the SUNYA revision of the Psychosomatic Symptom Checklist. J Behav Med. 1984;7(2):247-257. doi:10.1007/BF00845390
27. Weathers F, Litz B, Herman D, Huska J, Keane T. The PTSD Checklist (PCL): reliability, validity, and diagnostic utility. Accessed August 25, 2022. https://www.researchgate.net/publication/291448760_The_PTSD_Checklist_PCL_Reliability_validity_and_diagnostic_utility
28. Derogatis L. Brief Symptom Inventory-18 (BSI-18): Administration, Scoring, and Procedure Manual. Ed 3 ed. National Computer Systems; 2000.
29. Stamps BW, Lyon WJ, Irvin AP, Kelley-Loughnane N, Goodson MS. A pilot study of the effect of deployment on the gut microbiome and traveler´s diarrhea susceptibility. Front Cell Infect Microbiol. 2020;10:589297. doi:10.3389/fcimb.2020.589297
30. Kim HJ, Camilleri M, McKinzie S, et al. A randomized controlled trial of a probiotic, VSL#3, on gut transit and symptoms in diarrhoea-predominant irritable bowel syndrome. Aliment Pharmacol Ther. 2003;17(7):895-904. doi:10.1046/j.1365-2036.2003.01543.x
31. Kim HJ, Vazquez Roque MI, Camilleri M, et al. A randomized controlled trial of a probiotic combination VSL# 3 and placebo in irritable bowel syndrome with bloating. Neurogastroenterol Motil. 2005;17(5):687-696. doi:10.1111/j.1365-2982.2005.00695.x32. Michail S, Kenche H. Gut microbiota is not modified by randomized, double-blind, placebo-controlled trial of vsl#3 in diarrhea-predominant irritable bowel syndrome. Probiotics Antimicrob Proteins. 2011;3(1):1-7. doi:10.1007/s12602-010-9059-y
33. Moayyedi P, Ford AC, Talley NJ, et al. The efficacy of probiotics in the treatment of irritable bowel syndrome: a systematic review. Gut. 2010;59(3):325-332. doi:10.1136/gut.2008.167270
34. Ford AC, Harris LA, Lacy BE, Quigley EMM, Moayyedi P. Systematic review with meta-analysis: the efficacy of prebiotics, probiotics, synbiotics and antibiotics in irritable bowel syndrome. Aliment Pharmacol Ther. 2018;48(10):1044-1060. doi:10.1111/apt.15001
35. Dale HF, Rasmussen SH, Asiller OO, Lied GA. Probiotics in irritable bowel syndrome: an up-to-date systematic review. Nutrients. 2019;11(9). doi:10.3390/nu11092048
36. Didari T, Mozaffari S, Nikfar S, Abdollahi M. Effectiveness of probiotics in irritable bowel syndrome: Updated systematic review with meta-analysis. World J Gastroenterol. 2015;21(10):3072-84. doi:10.3748/wjg.v21.i10.3072
37. Hungin APS, Mitchell CR, Whorwell P, et al. Systematic review: probiotics in the management of lower gastrointestinal symptoms—an updated evidence-based international consensus. Aliment Pharmacol Ther. 2018;47(8):1054-1070. doi:10.1111/apt.14539
38. Niu HL, Xiao JY. The efficacy and safety of probiotics in patients with irritable bowel syndrome: evidence based on 35 randomized controlled trials. Int J Surg. 2020;75:116-127. doi:10.1016/j.ijsu.2020.01.142
39. Wong RK, Yang C, Song GH, Wong J, Ho KY. Melatonin regulation as a possible mechanism for probiotic (VSL#3) in irritable bowel syndrome: a randomized double-blinded placebo study. Dig Dis Sci. 2015;60(1):186-194. doi:10.1007/s10620-014-3299-8
40. Ford AC, Moayyedi P, Lacy BE, et al. American College of Gastroenterology monograph on the management of irritable bowel syndrome and chronic idiopathic constipation. Am J Gastroenterol. 2014;109(suppl 1):S2-26; quiz S27. doi: 10.1038/ajg.2014.187
41. Simren M, Barbara G, Flint HJ, et al. Intestinal microbiota in functional bowel disorders: a Rome foundation report. Gut. 2013;62(1):159-76. doi:10.1136/gutjnl-2012-302167
42. Ki Cha B, Mun Jung S, Hwan Choi C, et al. The effect of a multispecies probiotic mixture on the symptoms and fecal microbiota in diarrhea-dominant irritable bowel syndrome: a randomized, double-blind, placebo-controlled trial. J Clin Gastroenterol. 2012;46(3):220-7. doi:10.1097/MCG.0b013e31823712b1
43. Connell M, Shin A, James-Stevenson T, Xu H, Imperiale TF, Herron J. Systematic review and meta-analysis: Efficacy of patented probiotic, VSL#3, in irritable bowel syndrome. Neurogastroenterol Motil. 2018;30(12):e13427. doi:10.1111/nmo.13427
44. Lyra A, Hillila M, Huttunen T, et al. Irritable bowel syndrome symptom severity improves equally with probiotic and placebo. World J Gastroenterol. 2016;22(48):10631-10642. doi:10.3748/wjg.v22.i48.10631
45. Sanders ME, Guarner F, Guerrant R, et al. An update on the use and investigation of probiotics in health and disease. Gut. 2013;62(5):787-796. doi:10.1136/gutjnl-2012-302504
46. Tuteja AK. Deployment-associated functional gastrointestinal disorders: do we know the etiology? Dig Dis Sci. 2011;56(11):3109-3111. doi:10.1007/s10620-011-1856-y
About 700,000 US military personnel were deployed in Operation Desert Storm (August 1990 to March 1991).1 Almost 30 years since the war, a large number of these veterans continue to experience a complex of symptoms of unknown etiology called Gulf War illness (GWI), which significantly affects health and quality of life (QOL). The lack of clear etiology of the illness has impaired research to find specific treatments and has further exacerbated the stress among veterans. GWI typically includes a mixture of chronic headache, cognitive difficulties, widespread pain, unexplained fatigue, memory and concentration problems, as well as chronic respiratory and gastrointestinal (GI) symptoms.2 Abdominal pain and alteration of bowel habits are also symptoms typical of irritable bowel syndrome (IBS). It has been estimated that IBS occurs in up to 30% of Gulf War veterans.3
The etiology of IBS is unknown. Possible mechanisms include visceral hypersensitivity, altered gut motor function, aberrant brain-gut interaction, and psychological factors, perhaps with a genetic predisposition.4 Gastroenteritis has been reported as a triggering mechanism in up to one-third of patients with IBS.5 Gastroenteritis can alter the gut microbiota and has been reported to be a significant risk factor for the development of IBS.6 In one study of Operation Desert Shield soldiers, > 50% of military personnel developed acute gastroenteritis while on duty.7 A high prevalence of extra-intestinal symptoms also has been reported, including fatigue, headache, joint pains, and anxiety, in Gulf War veterans with IBS. These extra-intestinal symptoms of IBS are consistent with the reported GWI symptoms. Change in gut microbiota also has been associated with many of the extra-intestinal symptoms of IBS, especially fatigue.8,9 Gut microbiota are known to change with travel, stress, and a change in diet, all potential factors that are relevant to Gulf War veterans. This would suggest that an imbalance in the gut microbiota, ie, dysbiosis, may play a role in the pathogenesis of both IBS and GWI. Dysbiosis could be a risk factor for or alternatively a consequence of GWI.
A systematic review highlighted the heterogeneity of the gut microbiota in patients with IBS.10 Overall, Enterobacteriaceae, Lactobacillaceae, and Bacteroides were increased, whereas Clostridiales, Faecalibacterium, and Bifidobacterium were decreased in patients with IBS compared with controls. Gut microbiota also has been associated with cognitive changes, anxiety, and depression—symptoms associated with IBS and are part of the GWI.
If altered gut microbiota contributes to the etiopathogenesis of IBS, its restoration of with probiotics should help. Probiotics are live organisms that when ingested may improve health by promoting the growth of naturally occurring flora and establishing a healthy gut flora. Probiotics have several mechanisms of actions. Probiotics work in the lumen of the gut by producing antibacterial molecules and enhancing the mucosal barrier.11 Probiotics also may produce metabolic compounds that alter the intestinal microbiota and improve intestinal barrier function.12 Probiotics also have been shown to activate receptors in the enteric nervous system with the potential to promote pain relief in the setting of visceral hyperalgesia.13,14 The anti-inflammatory properties of probiotics potentially could modulate the basic pathophysiology of IBS and improve motility, visceral hypersensitivity, and brain-gut interaction.15 Furthermore, significant gut dysbiosis has been shown with GWI; suggesting that probiotics may have a role in its management.16,17
Probiotics have not been studied in Gulf War veterans with IBS. We performed a prospective, double-blind placebo-controlled study to determine the efficacy of a commercially available probiotic containing 8 strains of bacteria (De Simone Formulation; formally known as VSL#3 and Visbiome) on symptoms of IBS and GWI. This probiotic was selected as the overall literature suggested benefit of combination probiotics in IBS, and VSL#3 has been shown to be efficacious in ulcerative colitis and microscopic colitis.18-20
Methods
Veterans who served in Operation Desert Storm (August 1990 to March 1991) and enrolled at the George E. Wahlen Veterans Affairs (VA) Medical Center (GEWVAMC), Salt Lake City, Utah, were eligible for the study. The inclusion criteria were: veterans aged ≥ 35 years; ≥ 2 nonintestinal GWI symptoms (eg, fatigue, joint pains, insomnia, general stiffness, and headache); IBS diagnosis based on the Rome III criteria; IBS symptoms > 6 months; normal gross appearance of the colonic mucosa; negative markers for celiac disease and inflammatory bowel disease (IBD); normal thyroid function; and serum calcium levels.21 Those who had a clinically significant cardiac, pulmonary, hepatic or renal dysfunction; history of/or presence of systemic malignancy; current evidence of celiac disease or IBD; unstable/significant psychiatric disease; recent change in GI medications; current pregnancy; or use of antibiotics or probiotics within the past 1 month were excluded. Subjects were enrolled from a list of veterans with GWI from the GEWVAMC Gulf War registry; referrals to gastroenterology clinics for IBS from internal medicine clinics; and posted advertisements.
Protocol
After written informed consent was obtained, each veteran was verified to have IBS and ≥ 2 GWI symptoms. All veterans had the following tests and panels: complete blood count, erythrocyte sedimentation rate, serum comprehensive metabolic panel, thyroid-stimulating hormone, tissue transglutaminase, stool test for ova and parasite, giardia antigen, and clostridia toxins to exclude organic cause of GI symptoms. Colonoscopy was performed in all veterans to exclude IBD, and to rule out microscopic or lymphocytic colitis.
Randomization was computer generated and maintained by the study pharmacist so that study personnel and patients were blinded to the trial groups. All investigators were blinded and allocation was concealed. The medication was supplied in a numbered container by the pharmacist after patient enrollment. After a 2-week run-in period, veterans were randomized (1:1) to receive either 1 sachet of probiotic (De Simone Formulation; formally known as VSL#3 and Visbiome) or placebo once daily for 8 weeks.
Each probiotic packet contains 900 billion probiotic bacteria per sachet.11 This formulation contained 8 viable strains of bacteria: 4 strains of Lactobacillus (L acidophilus, L plantarum, L paracasei, L delbrueckii subsp. bulgaricus); 3 strains of Bifidobacteria (Bifidobacterium breve, B lactis, B infantis); and 1 strain of Streptococcus thermophilus. This formulation had been commercialized and studied as VSL#3 and is currently available in the United States under the Visbiome trade name. While branding changed during the study, the formulation did not. The investigational medicine (VSL#3, Visbiome, and placebo) were shipped from the manufacturer Dupont/Danisco in Madison, Wisconsin. The subjects received placebo or probiotic (VSL#3/Visbiome) and both were identical in appearance. The medication was supplied in a numbered container by the pharmacist after patient enrollment.
Measures
Veterans completed the bowel disease questionnaire to record baseline bowel habits.22 All veterans recorded daily bowel symptoms to confirm the presence of IBS during the 2-week pretreatment period, at baseline, and at the end of the 8-week treatment. The symptoms assessed included severity of abdominal pain (0, none to 100, severe); severity of bloating (0, none to 100, severe); stool frequency; Bristol stool scale (1, very hard to 7, watery); severity of diarrhea (0, none to 100, severe); severity of constipation (0, none to 100, severe); satisfaction with bowel habits (0, none to 100, severe); and IBS affecting or interfering with life (0, none to 100, severe). The bowel symptom score is the sum of the 5 symptom scores.23,24
IBS-specific QOL (IBS-QOL) was recorded at baseline and at the end of treatment.25 The IBS-QOL consists of a 34-item validated disease-specific questionnaire that measures 8 domains relevant to subjects with IBS: dysphoria, interference with activity, body image, health worry, food avoidance, social reaction, sexual life, and relationships. We used the Somatic Symptom Checklist to detect the following extra-intestinal symptoms that are common among veterans with GWI: headache, backache, wheeziness, insomnia, bad breath, fatigue, general stiffness, dizziness, weakness, sensitivity to hot and cold, palpitation, and tightness in chest. Subjects rated symptoms on a scale of 1 to 5: how often (1, none; 2, monthly; 3, once weekly; 4, several times weekly; 5, daily), and how bothersome (1, not at all to 5, extremely).26
Subjects completed the Posttraumatic Stress Disorder (PTSD) Checklist–Military, which is specific to military experience with 17 items on a 1 to 5 scale (1, not at all to 5, extremely). Scores were summed to produce a total symptom severity score (range, 17-85).27 Subjects also completed the Brief Symptom Inventory 18 (BSI-18) during the baseline evaluation.28 BSI-18 measures subjects’ reported overall psychological distress. It assesses 3 symptoms dimensions (somatization, depression, and anxiety) and a global severity index. The raw scores were transferred to normative T scores based on samples of nonpatient normal men and women.
Symptom data were compared after 8 weeks of treatment. The primary study endpoint was change in bowel symptom score. The secondary endpoints were mean change in symptoms, QOL, extra-intestinal symptoms, and PTSD score. The study was approved by the Salt Lake City Veterans Affairs Medical Center and the University of Utah Institutional Review Board and registered in ClinicalTrials.gov (NCT03078530).
Statistical Methods
Comparisons of the probiotic vs placebo groups for demographic variable were analyzed using a 2-sample t test for continuous variables, and with a χ2 test or Fisher exact test for categorical variables. The primary and secondary outcome variables were recorded daily for 2 weeks as pretreatment baseline and for 2 weeks at the end of treatment. These symptoms were recorded as ordered categorical variables, which were then averaged across the week to produce a continuous measurement for statistical analysis. For the primary outcome of GI symptoms, posttreatment comparisons were made between the study groups using a 2-sample t test of the baseline vs posttreatment values. All P values were calculated for 2-sided comparisons. The planned sample size in our study protocol was to recruit 40 individuals per group in order to achieve 80% power to detect a 30% improvement between baseline and end of treatment in the primary bowel symptom score. This study recruited 53 subjects. With this sample size, the study had 80% power to detect a 0.8 SD in any of the outcomes.
Results
We screened 101 veterans with IBS and GWI; 39 veterans did not fulfill the inclusion/exclusion criteria, 22 declined to participate or did not complete the screening questionnaires and tests, and 9 were lost to follow-up. Sixty-two participants were randomized in a double-blind placebo-controlled study design; 9 dropped out before the end of the study. Data were analyzed from 53 veterans who completed the study, 29 in the placebo group and 24 in the probiotic group (Figure 1). The cohort was primarily male with a mean (SD) age of 55 (8) years (range, 42-73) (Table 1).
Overall, the treatment was well tolerated. All subjects were contacted every 2 weeks during the study to check for adverse effects, but no serious events were reported. There were no differences at baseline in any of the BSI-18 subscale scores in veterans between the groups. There was a greater mean (SEM) improvement of diarrhea severity in the probiotic group compared with the placebo group: 18 (6), a 31% improvement, vs 6 (5), a 13% improvement, respectively; however, the difference was not statistically significance (P = .13) (Table 2). There also was a greater mean (SEM) improvement in satisfaction of bowel habits in the probiotic group compared with the placebo group: 16 (7), a 35% improvement vs 4 (9), an 8% worsening; this also was not statistically significant (P = .09). There was no difference in the change of IBS-QOL before and after treatment in either group (Figure 2). There was no improvement in any of the symptoms of GWI (all P ≥ .06) (Appendix).
Discussion
GWI is a complex multisystem illness of unknown etiology. There was high prevalence of diarrhea during deployment, and veterans were exposed to several physical, environmental, and mental stresses of the war.3 A change in gut microbiota can occur during deployment due to diet changes, environmental and physical stress, and GI infections.29 These changes would suggest that manipulation of gut microbiota might offer a new modality of treatment of IBS and GWI. We evaluated the effect of a high-potency multistrain probiotic in veterans with IBS and GWI. We did not detect any statistically significant differences between the probiotic and placebo groups on bowel symptom score and individual symptoms of IBS and on QOL. Also, there was no improvement for the other symptoms of GWI. To our knowledge, this is the first study evaluating the effect of probiotics in veterans with IBS and GWI. Our results are consistent with the literature on probiotics and IBS.
The probiotic formulation used in our study has been evaluated in patients with IBS previously. Kim and colleagues found that after 8 weeks of treatment of patients with diarrhea-predominant IBS with VSL#3, there was improvement in bloating, but no effect was found on abdominal pain, gas, or urgency.30 A subsequent study by the same investigators on patients with all types of IBS found that VSL#3 showed no effect on abdominal pain, stool frequency and consistency, or on bloating, but there was improvement in flatulence.31 Another study that evaluated the effect of VSL#3 on symptoms of diarrhea-predominant IBS and QOL found improvement in IBS symptoms from baseline in both the probiotic and the placebo groups, but the difference between the 2 groups was not statistically significant.32 Similarly, Wong and colleagues performed a double-blind, placebo-controlled mechanistic study to evaluate the effect of VSL#3. They found improvement in bowel symptom score, abdominal pain intensity, and satisfaction with bowel habits with both the VSL#3 and placebo group but similar to our study, the differences were not statistically significant.
Several reviews have evaluated the efficacy of probiotics for IBS. A 2010 review found evidence that probiotics trended toward improved IBS symptoms compared with placebo.33 The 2014 follow-up by the same authors demonstrated that overall, probiotics improved global symptoms of IBS and multistrain probiotics were more effective.20 A third meta-analysis from the same group found evidence that multistrain probiotics seemed to have a beneficial effect but could not definitively conclude that probiotics are efficacious in improving IBS symptoms.34 Other authors also have seen inconsistent effects of probiotics compared with placebo on global symptoms, abdominal pain, and bloating after performing systematic reviews of the literature.35-38 Although several reviews support that multistrain probiotics are more effective, they fail to conclude which combinations are more efficacious.
The effect of probiotics on QOL has not been investigated by many studies.37 In our study, we did not find significant improvement in QOL in the probiotic group, which is in line with 2 previous studies that showed no effect on IBS QOL of VSL#3 vs placebo.32,39 Most of the research reports that multistrain probiotics are more effective than using a single strain.34,35,40Bifidobacterium and Lactobacillus are the most commonly used bacteria in the multistrain probiotics that have shown their positive effect on IBS.35,41 The probiotic used in our study contained other species along with these 2 microorganisms.
The dose and duration of treatment of probiotics also has been debated. In one meta-analysis, the investigators found that studies of ≥ 8 weeks were more likely to show a positive effect; 4 of the 7 studies with statistically significant improvement in IBS symptoms were longer than 8 weeks.35 However, another meta-analysis based on 35 randomized controlled trials found that there was not a statistically significant difference between groups treated for > 4 weeks vs < 4 weeks.42 In addition, another meta-analysis of VSL#3 on IBS in children and adults also found no difference in results based on the duration of treatment of probiotics.43 Similar to our study, 3 other studies of VSL#3 treated patients for 8 weeks and found no statistically significant effect.30-32 In the past, VSL#3 has been used at dosages of 450 or 900 billion bacteria per day.
An individual’s response to probiotics may depend on the subtype of IBS. However, most of the studies, like ours, included groups of all subtypes. It may be that probiotics are more effective in patients with moderate-to-severe symptoms. Most of our patients had milder symptoms, and we cannot discount how subjects with more severe disease may have responded to the drug. Interestingly, one study demonstrated that Lactobacillus was more effective in patients with moderately severe abdominal pain compared with mild symptoms.44
In our study, the probiotic did not improve PTSD symptoms or other extra-intestinal symptoms common in IBS and GWI. Similar to our study, Wong and colleagues did not find significant improvement of psychological and sleep scores after treatment with VSL#3.6 Similarly, there is evidence that alteration in gut microbiota is associated with health and diseases, but what specific alterations occur and whether they can be improved with probiotics remains unknown.45
Limitations
The inconsistent response to probiotics in various studies may be due to IBS heterogeneity. Furthermore, there are demographic differences between Gulf War veterans and patients enrolled in other studies: Gulf War veterans are predominantly male, many were deployed abroad and had a history of gastroenteritis during deployment, and were exposed to stressful situations.46 These factors may be involved in triggering or maintaining IBS in Gulf War veterans. A further limitation of our randomized trial is the relatively small sample size.
Conclusions
This study did not demonstrate statistically significant improvement in symptoms of IBS or improvement in QOL after treatment with a multistrain probiotic. We also did not find any improvement in symptoms of GWI or PTSD. There was no difference in psychological scores between the placebo and treatment groups, and it is unlikely that psychological factors confounded the response to treatment in this study.
The effectiveness of a probiotic may depend on the baseline gut microbiome of the individual and depend on the strain, amount, and frequency of bacteria used. A lack of response of the probiotics does not exclude gut viruses and fungi having a role in exacerbating GWI symptoms. It is also possible that the bacteria present or the dose of the probiotic used was not sufficient to improve symptoms. So far, the definitive benefit of probiotics has been demonstrated for only a few preparations, and none are approved by the US Food and Drug Administration for any disease. More research is needed to determine whether probiotics have any role in the treatment of IBS and GWI.
Acknowledgments
AKT received grant support from the US Department of Veterans Affairs and the US Department of Defense (W81XWH-10-1-0593, W81XWH-15-1-0636). We thank Keith G. Tolman, MD, for assistance in editing the initial proposal and for periodic consultation. We thank the manufacturer of the probiotic for supplying the active drug and the placebo. The manufacture of the probiotic had no role in the design and conduct of the study, analysis and interpretation of the data, and in the preparation of the manuscript.
About 700,000 US military personnel were deployed in Operation Desert Storm (August 1990 to March 1991).1 Almost 30 years since the war, a large number of these veterans continue to experience a complex of symptoms of unknown etiology called Gulf War illness (GWI), which significantly affects health and quality of life (QOL). The lack of clear etiology of the illness has impaired research to find specific treatments and has further exacerbated the stress among veterans. GWI typically includes a mixture of chronic headache, cognitive difficulties, widespread pain, unexplained fatigue, memory and concentration problems, as well as chronic respiratory and gastrointestinal (GI) symptoms.2 Abdominal pain and alteration of bowel habits are also symptoms typical of irritable bowel syndrome (IBS). It has been estimated that IBS occurs in up to 30% of Gulf War veterans.3
The etiology of IBS is unknown. Possible mechanisms include visceral hypersensitivity, altered gut motor function, aberrant brain-gut interaction, and psychological factors, perhaps with a genetic predisposition.4 Gastroenteritis has been reported as a triggering mechanism in up to one-third of patients with IBS.5 Gastroenteritis can alter the gut microbiota and has been reported to be a significant risk factor for the development of IBS.6 In one study of Operation Desert Shield soldiers, > 50% of military personnel developed acute gastroenteritis while on duty.7 A high prevalence of extra-intestinal symptoms also has been reported, including fatigue, headache, joint pains, and anxiety, in Gulf War veterans with IBS. These extra-intestinal symptoms of IBS are consistent with the reported GWI symptoms. Change in gut microbiota also has been associated with many of the extra-intestinal symptoms of IBS, especially fatigue.8,9 Gut microbiota are known to change with travel, stress, and a change in diet, all potential factors that are relevant to Gulf War veterans. This would suggest that an imbalance in the gut microbiota, ie, dysbiosis, may play a role in the pathogenesis of both IBS and GWI. Dysbiosis could be a risk factor for or alternatively a consequence of GWI.
A systematic review highlighted the heterogeneity of the gut microbiota in patients with IBS.10 Overall, Enterobacteriaceae, Lactobacillaceae, and Bacteroides were increased, whereas Clostridiales, Faecalibacterium, and Bifidobacterium were decreased in patients with IBS compared with controls. Gut microbiota also has been associated with cognitive changes, anxiety, and depression—symptoms associated with IBS and are part of the GWI.
If altered gut microbiota contributes to the etiopathogenesis of IBS, its restoration of with probiotics should help. Probiotics are live organisms that when ingested may improve health by promoting the growth of naturally occurring flora and establishing a healthy gut flora. Probiotics have several mechanisms of actions. Probiotics work in the lumen of the gut by producing antibacterial molecules and enhancing the mucosal barrier.11 Probiotics also may produce metabolic compounds that alter the intestinal microbiota and improve intestinal barrier function.12 Probiotics also have been shown to activate receptors in the enteric nervous system with the potential to promote pain relief in the setting of visceral hyperalgesia.13,14 The anti-inflammatory properties of probiotics potentially could modulate the basic pathophysiology of IBS and improve motility, visceral hypersensitivity, and brain-gut interaction.15 Furthermore, significant gut dysbiosis has been shown with GWI; suggesting that probiotics may have a role in its management.16,17
Probiotics have not been studied in Gulf War veterans with IBS. We performed a prospective, double-blind placebo-controlled study to determine the efficacy of a commercially available probiotic containing 8 strains of bacteria (De Simone Formulation; formally known as VSL#3 and Visbiome) on symptoms of IBS and GWI. This probiotic was selected as the overall literature suggested benefit of combination probiotics in IBS, and VSL#3 has been shown to be efficacious in ulcerative colitis and microscopic colitis.18-20
Methods
Veterans who served in Operation Desert Storm (August 1990 to March 1991) and enrolled at the George E. Wahlen Veterans Affairs (VA) Medical Center (GEWVAMC), Salt Lake City, Utah, were eligible for the study. The inclusion criteria were: veterans aged ≥ 35 years; ≥ 2 nonintestinal GWI symptoms (eg, fatigue, joint pains, insomnia, general stiffness, and headache); IBS diagnosis based on the Rome III criteria; IBS symptoms > 6 months; normal gross appearance of the colonic mucosa; negative markers for celiac disease and inflammatory bowel disease (IBD); normal thyroid function; and serum calcium levels.21 Those who had a clinically significant cardiac, pulmonary, hepatic or renal dysfunction; history of/or presence of systemic malignancy; current evidence of celiac disease or IBD; unstable/significant psychiatric disease; recent change in GI medications; current pregnancy; or use of antibiotics or probiotics within the past 1 month were excluded. Subjects were enrolled from a list of veterans with GWI from the GEWVAMC Gulf War registry; referrals to gastroenterology clinics for IBS from internal medicine clinics; and posted advertisements.
Protocol
After written informed consent was obtained, each veteran was verified to have IBS and ≥ 2 GWI symptoms. All veterans had the following tests and panels: complete blood count, erythrocyte sedimentation rate, serum comprehensive metabolic panel, thyroid-stimulating hormone, tissue transglutaminase, stool test for ova and parasite, giardia antigen, and clostridia toxins to exclude organic cause of GI symptoms. Colonoscopy was performed in all veterans to exclude IBD, and to rule out microscopic or lymphocytic colitis.
Randomization was computer generated and maintained by the study pharmacist so that study personnel and patients were blinded to the trial groups. All investigators were blinded and allocation was concealed. The medication was supplied in a numbered container by the pharmacist after patient enrollment. After a 2-week run-in period, veterans were randomized (1:1) to receive either 1 sachet of probiotic (De Simone Formulation; formally known as VSL#3 and Visbiome) or placebo once daily for 8 weeks.
Each probiotic packet contains 900 billion probiotic bacteria per sachet.11 This formulation contained 8 viable strains of bacteria: 4 strains of Lactobacillus (L acidophilus, L plantarum, L paracasei, L delbrueckii subsp. bulgaricus); 3 strains of Bifidobacteria (Bifidobacterium breve, B lactis, B infantis); and 1 strain of Streptococcus thermophilus. This formulation had been commercialized and studied as VSL#3 and is currently available in the United States under the Visbiome trade name. While branding changed during the study, the formulation did not. The investigational medicine (VSL#3, Visbiome, and placebo) were shipped from the manufacturer Dupont/Danisco in Madison, Wisconsin. The subjects received placebo or probiotic (VSL#3/Visbiome) and both were identical in appearance. The medication was supplied in a numbered container by the pharmacist after patient enrollment.
Measures
Veterans completed the bowel disease questionnaire to record baseline bowel habits.22 All veterans recorded daily bowel symptoms to confirm the presence of IBS during the 2-week pretreatment period, at baseline, and at the end of the 8-week treatment. The symptoms assessed included severity of abdominal pain (0, none to 100, severe); severity of bloating (0, none to 100, severe); stool frequency; Bristol stool scale (1, very hard to 7, watery); severity of diarrhea (0, none to 100, severe); severity of constipation (0, none to 100, severe); satisfaction with bowel habits (0, none to 100, severe); and IBS affecting or interfering with life (0, none to 100, severe). The bowel symptom score is the sum of the 5 symptom scores.23,24
IBS-specific QOL (IBS-QOL) was recorded at baseline and at the end of treatment.25 The IBS-QOL consists of a 34-item validated disease-specific questionnaire that measures 8 domains relevant to subjects with IBS: dysphoria, interference with activity, body image, health worry, food avoidance, social reaction, sexual life, and relationships. We used the Somatic Symptom Checklist to detect the following extra-intestinal symptoms that are common among veterans with GWI: headache, backache, wheeziness, insomnia, bad breath, fatigue, general stiffness, dizziness, weakness, sensitivity to hot and cold, palpitation, and tightness in chest. Subjects rated symptoms on a scale of 1 to 5: how often (1, none; 2, monthly; 3, once weekly; 4, several times weekly; 5, daily), and how bothersome (1, not at all to 5, extremely).26
Subjects completed the Posttraumatic Stress Disorder (PTSD) Checklist–Military, which is specific to military experience with 17 items on a 1 to 5 scale (1, not at all to 5, extremely). Scores were summed to produce a total symptom severity score (range, 17-85).27 Subjects also completed the Brief Symptom Inventory 18 (BSI-18) during the baseline evaluation.28 BSI-18 measures subjects’ reported overall psychological distress. It assesses 3 symptoms dimensions (somatization, depression, and anxiety) and a global severity index. The raw scores were transferred to normative T scores based on samples of nonpatient normal men and women.
Symptom data were compared after 8 weeks of treatment. The primary study endpoint was change in bowel symptom score. The secondary endpoints were mean change in symptoms, QOL, extra-intestinal symptoms, and PTSD score. The study was approved by the Salt Lake City Veterans Affairs Medical Center and the University of Utah Institutional Review Board and registered in ClinicalTrials.gov (NCT03078530).
Statistical Methods
Comparisons of the probiotic vs placebo groups for demographic variable were analyzed using a 2-sample t test for continuous variables, and with a χ2 test or Fisher exact test for categorical variables. The primary and secondary outcome variables were recorded daily for 2 weeks as pretreatment baseline and for 2 weeks at the end of treatment. These symptoms were recorded as ordered categorical variables, which were then averaged across the week to produce a continuous measurement for statistical analysis. For the primary outcome of GI symptoms, posttreatment comparisons were made between the study groups using a 2-sample t test of the baseline vs posttreatment values. All P values were calculated for 2-sided comparisons. The planned sample size in our study protocol was to recruit 40 individuals per group in order to achieve 80% power to detect a 30% improvement between baseline and end of treatment in the primary bowel symptom score. This study recruited 53 subjects. With this sample size, the study had 80% power to detect a 0.8 SD in any of the outcomes.
Results
We screened 101 veterans with IBS and GWI; 39 veterans did not fulfill the inclusion/exclusion criteria, 22 declined to participate or did not complete the screening questionnaires and tests, and 9 were lost to follow-up. Sixty-two participants were randomized in a double-blind placebo-controlled study design; 9 dropped out before the end of the study. Data were analyzed from 53 veterans who completed the study, 29 in the placebo group and 24 in the probiotic group (Figure 1). The cohort was primarily male with a mean (SD) age of 55 (8) years (range, 42-73) (Table 1).
Overall, the treatment was well tolerated. All subjects were contacted every 2 weeks during the study to check for adverse effects, but no serious events were reported. There were no differences at baseline in any of the BSI-18 subscale scores in veterans between the groups. There was a greater mean (SEM) improvement of diarrhea severity in the probiotic group compared with the placebo group: 18 (6), a 31% improvement, vs 6 (5), a 13% improvement, respectively; however, the difference was not statistically significance (P = .13) (Table 2). There also was a greater mean (SEM) improvement in satisfaction of bowel habits in the probiotic group compared with the placebo group: 16 (7), a 35% improvement vs 4 (9), an 8% worsening; this also was not statistically significant (P = .09). There was no difference in the change of IBS-QOL before and after treatment in either group (Figure 2). There was no improvement in any of the symptoms of GWI (all P ≥ .06) (Appendix).
Discussion
GWI is a complex multisystem illness of unknown etiology. There was high prevalence of diarrhea during deployment, and veterans were exposed to several physical, environmental, and mental stresses of the war.3 A change in gut microbiota can occur during deployment due to diet changes, environmental and physical stress, and GI infections.29 These changes would suggest that manipulation of gut microbiota might offer a new modality of treatment of IBS and GWI. We evaluated the effect of a high-potency multistrain probiotic in veterans with IBS and GWI. We did not detect any statistically significant differences between the probiotic and placebo groups on bowel symptom score and individual symptoms of IBS and on QOL. Also, there was no improvement for the other symptoms of GWI. To our knowledge, this is the first study evaluating the effect of probiotics in veterans with IBS and GWI. Our results are consistent with the literature on probiotics and IBS.
The probiotic formulation used in our study has been evaluated in patients with IBS previously. Kim and colleagues found that after 8 weeks of treatment of patients with diarrhea-predominant IBS with VSL#3, there was improvement in bloating, but no effect was found on abdominal pain, gas, or urgency.30 A subsequent study by the same investigators on patients with all types of IBS found that VSL#3 showed no effect on abdominal pain, stool frequency and consistency, or on bloating, but there was improvement in flatulence.31 Another study that evaluated the effect of VSL#3 on symptoms of diarrhea-predominant IBS and QOL found improvement in IBS symptoms from baseline in both the probiotic and the placebo groups, but the difference between the 2 groups was not statistically significant.32 Similarly, Wong and colleagues performed a double-blind, placebo-controlled mechanistic study to evaluate the effect of VSL#3. They found improvement in bowel symptom score, abdominal pain intensity, and satisfaction with bowel habits with both the VSL#3 and placebo group but similar to our study, the differences were not statistically significant.
Several reviews have evaluated the efficacy of probiotics for IBS. A 2010 review found evidence that probiotics trended toward improved IBS symptoms compared with placebo.33 The 2014 follow-up by the same authors demonstrated that overall, probiotics improved global symptoms of IBS and multistrain probiotics were more effective.20 A third meta-analysis from the same group found evidence that multistrain probiotics seemed to have a beneficial effect but could not definitively conclude that probiotics are efficacious in improving IBS symptoms.34 Other authors also have seen inconsistent effects of probiotics compared with placebo on global symptoms, abdominal pain, and bloating after performing systematic reviews of the literature.35-38 Although several reviews support that multistrain probiotics are more effective, they fail to conclude which combinations are more efficacious.
The effect of probiotics on QOL has not been investigated by many studies.37 In our study, we did not find significant improvement in QOL in the probiotic group, which is in line with 2 previous studies that showed no effect on IBS QOL of VSL#3 vs placebo.32,39 Most of the research reports that multistrain probiotics are more effective than using a single strain.34,35,40Bifidobacterium and Lactobacillus are the most commonly used bacteria in the multistrain probiotics that have shown their positive effect on IBS.35,41 The probiotic used in our study contained other species along with these 2 microorganisms.
The dose and duration of treatment of probiotics also has been debated. In one meta-analysis, the investigators found that studies of ≥ 8 weeks were more likely to show a positive effect; 4 of the 7 studies with statistically significant improvement in IBS symptoms were longer than 8 weeks.35 However, another meta-analysis based on 35 randomized controlled trials found that there was not a statistically significant difference between groups treated for > 4 weeks vs < 4 weeks.42 In addition, another meta-analysis of VSL#3 on IBS in children and adults also found no difference in results based on the duration of treatment of probiotics.43 Similar to our study, 3 other studies of VSL#3 treated patients for 8 weeks and found no statistically significant effect.30-32 In the past, VSL#3 has been used at dosages of 450 or 900 billion bacteria per day.
An individual’s response to probiotics may depend on the subtype of IBS. However, most of the studies, like ours, included groups of all subtypes. It may be that probiotics are more effective in patients with moderate-to-severe symptoms. Most of our patients had milder symptoms, and we cannot discount how subjects with more severe disease may have responded to the drug. Interestingly, one study demonstrated that Lactobacillus was more effective in patients with moderately severe abdominal pain compared with mild symptoms.44
In our study, the probiotic did not improve PTSD symptoms or other extra-intestinal symptoms common in IBS and GWI. Similar to our study, Wong and colleagues did not find significant improvement of psychological and sleep scores after treatment with VSL#3.6 Similarly, there is evidence that alteration in gut microbiota is associated with health and diseases, but what specific alterations occur and whether they can be improved with probiotics remains unknown.45
Limitations
The inconsistent response to probiotics in various studies may be due to IBS heterogeneity. Furthermore, there are demographic differences between Gulf War veterans and patients enrolled in other studies: Gulf War veterans are predominantly male, many were deployed abroad and had a history of gastroenteritis during deployment, and were exposed to stressful situations.46 These factors may be involved in triggering or maintaining IBS in Gulf War veterans. A further limitation of our randomized trial is the relatively small sample size.
Conclusions
This study did not demonstrate statistically significant improvement in symptoms of IBS or improvement in QOL after treatment with a multistrain probiotic. We also did not find any improvement in symptoms of GWI or PTSD. There was no difference in psychological scores between the placebo and treatment groups, and it is unlikely that psychological factors confounded the response to treatment in this study.
The effectiveness of a probiotic may depend on the baseline gut microbiome of the individual and depend on the strain, amount, and frequency of bacteria used. A lack of response of the probiotics does not exclude gut viruses and fungi having a role in exacerbating GWI symptoms. It is also possible that the bacteria present or the dose of the probiotic used was not sufficient to improve symptoms. So far, the definitive benefit of probiotics has been demonstrated for only a few preparations, and none are approved by the US Food and Drug Administration for any disease. More research is needed to determine whether probiotics have any role in the treatment of IBS and GWI.
Acknowledgments
AKT received grant support from the US Department of Veterans Affairs and the US Department of Defense (W81XWH-10-1-0593, W81XWH-15-1-0636). We thank Keith G. Tolman, MD, for assistance in editing the initial proposal and for periodic consultation. We thank the manufacturer of the probiotic for supplying the active drug and the placebo. The manufacture of the probiotic had no role in the design and conduct of the study, analysis and interpretation of the data, and in the preparation of the manuscript.
1. O’Shea EF, Cotter PD, Stanton C, Ross RP, Hill C. Production of bioactive substances by intestinal bacteria as a basis for explaining probiotic mechanisms: bacteriocins and conjugated linoleic acid. Int J Food Microbiol. 2012;152(3):189-205. doi:10.1016/j.ijfoodmicro.2011.05.025.
2. Kamiya T, Wang L, Forsythe P, et al. Inhibitory effects of Lactobacillus reuteri on visceral pain induced by colorectal distension in Sprague-Dawley rats. Gut. 2006;55(2):191-196. doi:10.1136/gut.2005.070987.
3. Verdu EF, Bercik P, Verma-Gandhu M, et al. Specific probiotic therapy attenuates antibiotic induced visceral hypersensitivity in mice. Gut. 2006;55(2):182-190. doi:10.1136/gut.2005.066100
4. Ford AC, Harris LA, Lacy BE, Quigley EMM, Moayyedi P. Systematic review with meta-analysis: the efficacy of prebiotics, probiotics, synbiotics and antibiotics in irritable bowel syndrome. Aliment Pharmacol Ther. 2018;48(10):1044-1060. doi:10.1111/apt.15001.
5. Niu HL, Xiao JY. The efficacy and safety of probiotics in patients with irritable bowel syndrome: Evidence based on 35 randomized controlled trials. Int J Surg. 2020;75:116-127. doi:10.1016/j.ijsu.2020.01.142.
6. Wong RK, Yang C, Song GH, Wong J, Ho KY. Melatonin regulation as a possible mechanism for probiotic (VSL#3) in irritable bowel syndrome: a randomized double-blinded placebo study. Dig Dis Sci. 2015;60(1):186-194. doi:10.1007/s10620-014-3299-8.
7. Hyams KC, Bourgeois AL, Merrell BR, et al. Diarrheal disease during Operation Desert Shield. N Engl J Med. 1991;325(20):1423-1428. doi:10.1056/NEJM199111143252006 8. Clancy RL, Gleeson M, Cox A, et al. Reversal in fatigued athletes of a defect in interferon gamma secretion after administration of Lactobacillus acidophilus. Br J Sports Med. 2006;40(4):351-354. doi:10.1136/bjsm.2005.024364
9. Sullivan A, Nord CE, Evengard B. Effect of supplement with lactic-acid producing bacteria on fatigue and physical activity in patients with chronic fatigue syndrome. Nutr J. 2009;8:4. doi:10.1186/1475-2891-8-4
10. Pittayanon R, Lau JT, Yuan Y, et al. Gut microbiota in patients with irritable bowel syndrome—a systematic review. Gastroenterology. 2019;157(1):97-108. doi:10.1053/j.gastro.2019.03.049
11. Rao RK, Samak G. Protection and restitution of gut barrier by probiotics: nutritional and clinical implications. Curr Nutr Food Sci. 2013;9(2):99-107. doi:10.2174/1573401311309020004
12. O´Shea EF, Cotter PD, Stanton C, Ross RP, Hill C. Production of bioactive substances by intestinal bacteria as a basis for explaining probiotic mechanisms: bacteriocins and conjugated linoleic acid. Int J Food Microbiol. 2012;152(3):189-205. doi:10.1016/j.ijfoodmicro.2011.05.025
13. Kamiya T, Wang L, Forsythe P, et al. Inhibitory effects of Lactobacillus reuteri on visceral pain induced by colorectal distension in Sprague-Dawley rats. Gut. 2006;55(2):191-196. doi:10.1136/gut.2005.070987
14. Verdu EF, Bercik P, Verma-Gandhu M, et al. Specific probiotic therapy attenuates antibiotic induced visceral hypersensitivity in mice. Gut. 2006;55(2):182-190. doi:10.1136/gut.2005.06610015. O´Mahony L, McCarthy J, Kelly P, et al. Lactobacillus and bifidobacterium in irritable bowel syndrome: symptom responses and relationship to cytokine profiles. Gastroenterology. 2005;128(3):541-551. doi:10.1053/j.gastro.2004.11.050
16. Alhasson F, Das S, Seth R, et al. Altered gut microbiome in a mouse model of Gulf War Illness causes neuroinflammation and intestinal injury via leaky gut and TLR4 activation. PLoS One. 2017;12(3):e0172914. doi:10.1371/journal.pone.0172914.17. Janulewicz PA, Seth RK, Carlson JM, et al. The gut-microbiome in Gulf War veterans: a preliminary report. Int J Environ Res Public Health. 2019;16(19). doi:10.3390/ijerph16193751
18. Dang X, Xu M, Liu D, Zhou D, Yang W. Assessing the efficacy and safety of fecal microbiota transplantation and probiotic VSL#3 for active ulcerative colitis: a systematic review and meta-analysis. PLoS One. 2020;15(3):e0228846. doi:10.1371/journal.pone.0228846
19. Ford AC, Quigley EM, Lacy BE, et al. Efficacy of prebiotics, probiotics, and synbiotics in irritable bowel syndrome and chronic idiopathic constipation: systematic review and meta-analysis. Am J Gastroenterol. 2014;109(10):1547-1561; quiz 1546, 1562. doi:10.1038/ajg.2014.202
20. Rohatgi S, Ahuja V, Makharia GK, et al. VSL#3 induces and maintains short-term clinical response in patients with active microscopic colitis: a two-phase randomised clinical trial. BMJ Open Gastroenterol. 2015;2(1):e000018. doi:10.1136/bmjgast-2014-000018
21. Longstreth GF, Thompson WG, Chey WD, Houghton LA, Mearin F, Spiller RC. Functional bowel disorders. Gastroenterology. 2006;130(5):1480-1491. doi:10.1053/j.gastro.2005.11.061
22. Talley NJ, Phillips SF, Melton J, 3rd, Wiltgen C, Zinsmeister AR. A patient questionnaire to identify bowel disease. Ann Intern Med. 1989;111(8):671-674. doi:10.7326/0003-4819-111-8-671
23. Bensoussan A, Talley NJ, Hing M, Menzies R, Guo A, Ngu M. Treatment of irritable bowel syndrome with Chinese herbal medicine: a randomized controlled trial. JAMA. 1998;280(18):1585-1589. doi:10.1001/jama.280.18.1585
24. Francis CY, Morris J, Whorwell PJ. The irritable bowel severity scoring system: a simple method of monitoring irritable bowel syndrome and its progress. Aliment Pharmacol Ther. 1997;11(2):395-402. doi:10.1046/j.1365-2036.1997.142318000.x
25. Patrick DL, Drossman DA, Frederick IO, DiCesare J, Puder KL. Quality of life in persons with irritable bowel syndrome: development and validation of a new measure. Dig Dis Sci. 1998;43(2):400-411. doi:10.1023/a:1018831127942
26. Attanasio V, Andrasik F, Blanchard EB, Arena JG. Psychometric properties of the SUNYA revision of the Psychosomatic Symptom Checklist. J Behav Med. 1984;7(2):247-257. doi:10.1007/BF00845390
27. Weathers F, Litz B, Herman D, Huska J, Keane T. The PTSD Checklist (PCL): reliability, validity, and diagnostic utility. Accessed August 25, 2022. https://www.researchgate.net/publication/291448760_The_PTSD_Checklist_PCL_Reliability_validity_and_diagnostic_utility
28. Derogatis L. Brief Symptom Inventory-18 (BSI-18): Administration, Scoring, and Procedure Manual. Ed 3 ed. National Computer Systems; 2000.
29. Stamps BW, Lyon WJ, Irvin AP, Kelley-Loughnane N, Goodson MS. A pilot study of the effect of deployment on the gut microbiome and traveler´s diarrhea susceptibility. Front Cell Infect Microbiol. 2020;10:589297. doi:10.3389/fcimb.2020.589297
30. Kim HJ, Camilleri M, McKinzie S, et al. A randomized controlled trial of a probiotic, VSL#3, on gut transit and symptoms in diarrhoea-predominant irritable bowel syndrome. Aliment Pharmacol Ther. 2003;17(7):895-904. doi:10.1046/j.1365-2036.2003.01543.x
31. Kim HJ, Vazquez Roque MI, Camilleri M, et al. A randomized controlled trial of a probiotic combination VSL# 3 and placebo in irritable bowel syndrome with bloating. Neurogastroenterol Motil. 2005;17(5):687-696. doi:10.1111/j.1365-2982.2005.00695.x32. Michail S, Kenche H. Gut microbiota is not modified by randomized, double-blind, placebo-controlled trial of vsl#3 in diarrhea-predominant irritable bowel syndrome. Probiotics Antimicrob Proteins. 2011;3(1):1-7. doi:10.1007/s12602-010-9059-y
33. Moayyedi P, Ford AC, Talley NJ, et al. The efficacy of probiotics in the treatment of irritable bowel syndrome: a systematic review. Gut. 2010;59(3):325-332. doi:10.1136/gut.2008.167270
34. Ford AC, Harris LA, Lacy BE, Quigley EMM, Moayyedi P. Systematic review with meta-analysis: the efficacy of prebiotics, probiotics, synbiotics and antibiotics in irritable bowel syndrome. Aliment Pharmacol Ther. 2018;48(10):1044-1060. doi:10.1111/apt.15001
35. Dale HF, Rasmussen SH, Asiller OO, Lied GA. Probiotics in irritable bowel syndrome: an up-to-date systematic review. Nutrients. 2019;11(9). doi:10.3390/nu11092048
36. Didari T, Mozaffari S, Nikfar S, Abdollahi M. Effectiveness of probiotics in irritable bowel syndrome: Updated systematic review with meta-analysis. World J Gastroenterol. 2015;21(10):3072-84. doi:10.3748/wjg.v21.i10.3072
37. Hungin APS, Mitchell CR, Whorwell P, et al. Systematic review: probiotics in the management of lower gastrointestinal symptoms—an updated evidence-based international consensus. Aliment Pharmacol Ther. 2018;47(8):1054-1070. doi:10.1111/apt.14539
38. Niu HL, Xiao JY. The efficacy and safety of probiotics in patients with irritable bowel syndrome: evidence based on 35 randomized controlled trials. Int J Surg. 2020;75:116-127. doi:10.1016/j.ijsu.2020.01.142
39. Wong RK, Yang C, Song GH, Wong J, Ho KY. Melatonin regulation as a possible mechanism for probiotic (VSL#3) in irritable bowel syndrome: a randomized double-blinded placebo study. Dig Dis Sci. 2015;60(1):186-194. doi:10.1007/s10620-014-3299-8
40. Ford AC, Moayyedi P, Lacy BE, et al. American College of Gastroenterology monograph on the management of irritable bowel syndrome and chronic idiopathic constipation. Am J Gastroenterol. 2014;109(suppl 1):S2-26; quiz S27. doi: 10.1038/ajg.2014.187
41. Simren M, Barbara G, Flint HJ, et al. Intestinal microbiota in functional bowel disorders: a Rome foundation report. Gut. 2013;62(1):159-76. doi:10.1136/gutjnl-2012-302167
42. Ki Cha B, Mun Jung S, Hwan Choi C, et al. The effect of a multispecies probiotic mixture on the symptoms and fecal microbiota in diarrhea-dominant irritable bowel syndrome: a randomized, double-blind, placebo-controlled trial. J Clin Gastroenterol. 2012;46(3):220-7. doi:10.1097/MCG.0b013e31823712b1
43. Connell M, Shin A, James-Stevenson T, Xu H, Imperiale TF, Herron J. Systematic review and meta-analysis: Efficacy of patented probiotic, VSL#3, in irritable bowel syndrome. Neurogastroenterol Motil. 2018;30(12):e13427. doi:10.1111/nmo.13427
44. Lyra A, Hillila M, Huttunen T, et al. Irritable bowel syndrome symptom severity improves equally with probiotic and placebo. World J Gastroenterol. 2016;22(48):10631-10642. doi:10.3748/wjg.v22.i48.10631
45. Sanders ME, Guarner F, Guerrant R, et al. An update on the use and investigation of probiotics in health and disease. Gut. 2013;62(5):787-796. doi:10.1136/gutjnl-2012-302504
46. Tuteja AK. Deployment-associated functional gastrointestinal disorders: do we know the etiology? Dig Dis Sci. 2011;56(11):3109-3111. doi:10.1007/s10620-011-1856-y
1. O’Shea EF, Cotter PD, Stanton C, Ross RP, Hill C. Production of bioactive substances by intestinal bacteria as a basis for explaining probiotic mechanisms: bacteriocins and conjugated linoleic acid. Int J Food Microbiol. 2012;152(3):189-205. doi:10.1016/j.ijfoodmicro.2011.05.025.
2. Kamiya T, Wang L, Forsythe P, et al. Inhibitory effects of Lactobacillus reuteri on visceral pain induced by colorectal distension in Sprague-Dawley rats. Gut. 2006;55(2):191-196. doi:10.1136/gut.2005.070987.
3. Verdu EF, Bercik P, Verma-Gandhu M, et al. Specific probiotic therapy attenuates antibiotic induced visceral hypersensitivity in mice. Gut. 2006;55(2):182-190. doi:10.1136/gut.2005.066100
4. Ford AC, Harris LA, Lacy BE, Quigley EMM, Moayyedi P. Systematic review with meta-analysis: the efficacy of prebiotics, probiotics, synbiotics and antibiotics in irritable bowel syndrome. Aliment Pharmacol Ther. 2018;48(10):1044-1060. doi:10.1111/apt.15001.
5. Niu HL, Xiao JY. The efficacy and safety of probiotics in patients with irritable bowel syndrome: Evidence based on 35 randomized controlled trials. Int J Surg. 2020;75:116-127. doi:10.1016/j.ijsu.2020.01.142.
6. Wong RK, Yang C, Song GH, Wong J, Ho KY. Melatonin regulation as a possible mechanism for probiotic (VSL#3) in irritable bowel syndrome: a randomized double-blinded placebo study. Dig Dis Sci. 2015;60(1):186-194. doi:10.1007/s10620-014-3299-8.
7. Hyams KC, Bourgeois AL, Merrell BR, et al. Diarrheal disease during Operation Desert Shield. N Engl J Med. 1991;325(20):1423-1428. doi:10.1056/NEJM199111143252006 8. Clancy RL, Gleeson M, Cox A, et al. Reversal in fatigued athletes of a defect in interferon gamma secretion after administration of Lactobacillus acidophilus. Br J Sports Med. 2006;40(4):351-354. doi:10.1136/bjsm.2005.024364
9. Sullivan A, Nord CE, Evengard B. Effect of supplement with lactic-acid producing bacteria on fatigue and physical activity in patients with chronic fatigue syndrome. Nutr J. 2009;8:4. doi:10.1186/1475-2891-8-4
10. Pittayanon R, Lau JT, Yuan Y, et al. Gut microbiota in patients with irritable bowel syndrome—a systematic review. Gastroenterology. 2019;157(1):97-108. doi:10.1053/j.gastro.2019.03.049
11. Rao RK, Samak G. Protection and restitution of gut barrier by probiotics: nutritional and clinical implications. Curr Nutr Food Sci. 2013;9(2):99-107. doi:10.2174/1573401311309020004
12. O´Shea EF, Cotter PD, Stanton C, Ross RP, Hill C. Production of bioactive substances by intestinal bacteria as a basis for explaining probiotic mechanisms: bacteriocins and conjugated linoleic acid. Int J Food Microbiol. 2012;152(3):189-205. doi:10.1016/j.ijfoodmicro.2011.05.025
13. Kamiya T, Wang L, Forsythe P, et al. Inhibitory effects of Lactobacillus reuteri on visceral pain induced by colorectal distension in Sprague-Dawley rats. Gut. 2006;55(2):191-196. doi:10.1136/gut.2005.070987
14. Verdu EF, Bercik P, Verma-Gandhu M, et al. Specific probiotic therapy attenuates antibiotic induced visceral hypersensitivity in mice. Gut. 2006;55(2):182-190. doi:10.1136/gut.2005.06610015. O´Mahony L, McCarthy J, Kelly P, et al. Lactobacillus and bifidobacterium in irritable bowel syndrome: symptom responses and relationship to cytokine profiles. Gastroenterology. 2005;128(3):541-551. doi:10.1053/j.gastro.2004.11.050
16. Alhasson F, Das S, Seth R, et al. Altered gut microbiome in a mouse model of Gulf War Illness causes neuroinflammation and intestinal injury via leaky gut and TLR4 activation. PLoS One. 2017;12(3):e0172914. doi:10.1371/journal.pone.0172914.17. Janulewicz PA, Seth RK, Carlson JM, et al. The gut-microbiome in Gulf War veterans: a preliminary report. Int J Environ Res Public Health. 2019;16(19). doi:10.3390/ijerph16193751
18. Dang X, Xu M, Liu D, Zhou D, Yang W. Assessing the efficacy and safety of fecal microbiota transplantation and probiotic VSL#3 for active ulcerative colitis: a systematic review and meta-analysis. PLoS One. 2020;15(3):e0228846. doi:10.1371/journal.pone.0228846
19. Ford AC, Quigley EM, Lacy BE, et al. Efficacy of prebiotics, probiotics, and synbiotics in irritable bowel syndrome and chronic idiopathic constipation: systematic review and meta-analysis. Am J Gastroenterol. 2014;109(10):1547-1561; quiz 1546, 1562. doi:10.1038/ajg.2014.202
20. Rohatgi S, Ahuja V, Makharia GK, et al. VSL#3 induces and maintains short-term clinical response in patients with active microscopic colitis: a two-phase randomised clinical trial. BMJ Open Gastroenterol. 2015;2(1):e000018. doi:10.1136/bmjgast-2014-000018
21. Longstreth GF, Thompson WG, Chey WD, Houghton LA, Mearin F, Spiller RC. Functional bowel disorders. Gastroenterology. 2006;130(5):1480-1491. doi:10.1053/j.gastro.2005.11.061
22. Talley NJ, Phillips SF, Melton J, 3rd, Wiltgen C, Zinsmeister AR. A patient questionnaire to identify bowel disease. Ann Intern Med. 1989;111(8):671-674. doi:10.7326/0003-4819-111-8-671
23. Bensoussan A, Talley NJ, Hing M, Menzies R, Guo A, Ngu M. Treatment of irritable bowel syndrome with Chinese herbal medicine: a randomized controlled trial. JAMA. 1998;280(18):1585-1589. doi:10.1001/jama.280.18.1585
24. Francis CY, Morris J, Whorwell PJ. The irritable bowel severity scoring system: a simple method of monitoring irritable bowel syndrome and its progress. Aliment Pharmacol Ther. 1997;11(2):395-402. doi:10.1046/j.1365-2036.1997.142318000.x
25. Patrick DL, Drossman DA, Frederick IO, DiCesare J, Puder KL. Quality of life in persons with irritable bowel syndrome: development and validation of a new measure. Dig Dis Sci. 1998;43(2):400-411. doi:10.1023/a:1018831127942
26. Attanasio V, Andrasik F, Blanchard EB, Arena JG. Psychometric properties of the SUNYA revision of the Psychosomatic Symptom Checklist. J Behav Med. 1984;7(2):247-257. doi:10.1007/BF00845390
27. Weathers F, Litz B, Herman D, Huska J, Keane T. The PTSD Checklist (PCL): reliability, validity, and diagnostic utility. Accessed August 25, 2022. https://www.researchgate.net/publication/291448760_The_PTSD_Checklist_PCL_Reliability_validity_and_diagnostic_utility
28. Derogatis L. Brief Symptom Inventory-18 (BSI-18): Administration, Scoring, and Procedure Manual. Ed 3 ed. National Computer Systems; 2000.
29. Stamps BW, Lyon WJ, Irvin AP, Kelley-Loughnane N, Goodson MS. A pilot study of the effect of deployment on the gut microbiome and traveler´s diarrhea susceptibility. Front Cell Infect Microbiol. 2020;10:589297. doi:10.3389/fcimb.2020.589297
30. Kim HJ, Camilleri M, McKinzie S, et al. A randomized controlled trial of a probiotic, VSL#3, on gut transit and symptoms in diarrhoea-predominant irritable bowel syndrome. Aliment Pharmacol Ther. 2003;17(7):895-904. doi:10.1046/j.1365-2036.2003.01543.x
31. Kim HJ, Vazquez Roque MI, Camilleri M, et al. A randomized controlled trial of a probiotic combination VSL# 3 and placebo in irritable bowel syndrome with bloating. Neurogastroenterol Motil. 2005;17(5):687-696. doi:10.1111/j.1365-2982.2005.00695.x32. Michail S, Kenche H. Gut microbiota is not modified by randomized, double-blind, placebo-controlled trial of vsl#3 in diarrhea-predominant irritable bowel syndrome. Probiotics Antimicrob Proteins. 2011;3(1):1-7. doi:10.1007/s12602-010-9059-y
33. Moayyedi P, Ford AC, Talley NJ, et al. The efficacy of probiotics in the treatment of irritable bowel syndrome: a systematic review. Gut. 2010;59(3):325-332. doi:10.1136/gut.2008.167270
34. Ford AC, Harris LA, Lacy BE, Quigley EMM, Moayyedi P. Systematic review with meta-analysis: the efficacy of prebiotics, probiotics, synbiotics and antibiotics in irritable bowel syndrome. Aliment Pharmacol Ther. 2018;48(10):1044-1060. doi:10.1111/apt.15001
35. Dale HF, Rasmussen SH, Asiller OO, Lied GA. Probiotics in irritable bowel syndrome: an up-to-date systematic review. Nutrients. 2019;11(9). doi:10.3390/nu11092048
36. Didari T, Mozaffari S, Nikfar S, Abdollahi M. Effectiveness of probiotics in irritable bowel syndrome: Updated systematic review with meta-analysis. World J Gastroenterol. 2015;21(10):3072-84. doi:10.3748/wjg.v21.i10.3072
37. Hungin APS, Mitchell CR, Whorwell P, et al. Systematic review: probiotics in the management of lower gastrointestinal symptoms—an updated evidence-based international consensus. Aliment Pharmacol Ther. 2018;47(8):1054-1070. doi:10.1111/apt.14539
38. Niu HL, Xiao JY. The efficacy and safety of probiotics in patients with irritable bowel syndrome: evidence based on 35 randomized controlled trials. Int J Surg. 2020;75:116-127. doi:10.1016/j.ijsu.2020.01.142
39. Wong RK, Yang C, Song GH, Wong J, Ho KY. Melatonin regulation as a possible mechanism for probiotic (VSL#3) in irritable bowel syndrome: a randomized double-blinded placebo study. Dig Dis Sci. 2015;60(1):186-194. doi:10.1007/s10620-014-3299-8
40. Ford AC, Moayyedi P, Lacy BE, et al. American College of Gastroenterology monograph on the management of irritable bowel syndrome and chronic idiopathic constipation. Am J Gastroenterol. 2014;109(suppl 1):S2-26; quiz S27. doi: 10.1038/ajg.2014.187
41. Simren M, Barbara G, Flint HJ, et al. Intestinal microbiota in functional bowel disorders: a Rome foundation report. Gut. 2013;62(1):159-76. doi:10.1136/gutjnl-2012-302167
42. Ki Cha B, Mun Jung S, Hwan Choi C, et al. The effect of a multispecies probiotic mixture on the symptoms and fecal microbiota in diarrhea-dominant irritable bowel syndrome: a randomized, double-blind, placebo-controlled trial. J Clin Gastroenterol. 2012;46(3):220-7. doi:10.1097/MCG.0b013e31823712b1
43. Connell M, Shin A, James-Stevenson T, Xu H, Imperiale TF, Herron J. Systematic review and meta-analysis: Efficacy of patented probiotic, VSL#3, in irritable bowel syndrome. Neurogastroenterol Motil. 2018;30(12):e13427. doi:10.1111/nmo.13427
44. Lyra A, Hillila M, Huttunen T, et al. Irritable bowel syndrome symptom severity improves equally with probiotic and placebo. World J Gastroenterol. 2016;22(48):10631-10642. doi:10.3748/wjg.v22.i48.10631
45. Sanders ME, Guarner F, Guerrant R, et al. An update on the use and investigation of probiotics in health and disease. Gut. 2013;62(5):787-796. doi:10.1136/gutjnl-2012-302504
46. Tuteja AK. Deployment-associated functional gastrointestinal disorders: do we know the etiology? Dig Dis Sci. 2011;56(11):3109-3111. doi:10.1007/s10620-011-1856-y
Spontaneous ecchymoses
A 65-YEAR-OLD WOMAN was brought into the emergency department by her daughter for spontaneous bruising, fatigue, and weakness of several weeks’ duration. She denied taking any medications or illicit drugs and had not experienced any falls or trauma. On a daily basis, she smoked 5 to 7 cigarettes and drank 6 or 7 beers, as had been her custom for several years. The patient lived alone and was grieving the death of her beloved dog, who had died a month earlier. She reported that since the death of her dog, her diet, which hadn’t been especially good to begin with, had deteriorated; it now consisted of beer and crackers.
On admission, she was mildly tachycardic (105 beats/min) with a blood pressure of 125/66 mm Hg. Physical examination revealed a frail-appearing woman who was in no acute distress but was unable to stand without assistance. She had diffuse ecchymoses and perifollicular, purpuric, hyperkeratotic papules and plaques on both of her legs (FIGURES 1A and 1B). In addition, she had faint perifollicular purpuric macules on her upper back. An oral examination revealed poor dentition.
A punch biopsy was performed on her leg, and it revealed noninflammatory dermal hemorrhage without evidence of vasculitis or vasculopathy.
WHAT IS YOUR DIAGNOSIS?
HOW WOULD YOU TREAT THIS PATIENT?
Diagnosis: Scurvy
Based on the patient’s appearance and her dietary history, we suspected scurvy, so a serum vitamin C level was ordered. The results took several days to return. In the meantime, additional lab work revealed hyponatremia (sodium, 129 mmol/L; normal range, 135-145 mmol/L), hypokalemia (potassium, 3 mmol/L; normal range, 3.5-5.2 mmol/L), hypophosphatemia (phosphorus, 2.3 mg/dL; normal range, 2.8-4.5 mg/dL); low serum vitamin D (6 ng/mL; normal range, 20-40 ng/mL); and macrocytic anemia (hemoglobin, 7.4 g/dL; normal range, 11-18 g/dL) with a mean corpuscular volume of 101.1 fL (normal range, 80-100 fL). Her iron panel showed normal serum iron and total iron binding capacity with a normal ferritin level (294 ng/mL; normal range, 30-300 ng/mL). A peripheral blood smear test uncovered mild anisocytosis and polychromasia, with no schistocytes. A fecal immunochemical test was negative.
Several days after admission, the results of the patient’s vitamin C test came back. Her levels were undetectable (< 5 µmol/L; normal range, 11-23 µmol/L), confirming that the patient had scurvy.
A health hazard to marinersthat is still around today
Scurvy is a condition that arises from a deficiency of vitamin C, or ascorbic acid. The first known case of scurvy was in 1550 BC.1 Hippocrates termed the condition “ileos ematitis” and stated that “the mouth feels bad; the gums are detached from the teeth; blood runs from the nostrils … ulcerations on the legs … skin is thin.”1 Scurvy was a major health hazard of mariners between the 15thand 18th centuries.2 Today, the deficiency is uncommon in industrialized countries because there are many sources of vitamin C available through diet and vitamin supplements.3 In the United States, the prevalence of vitamin C deficiency is approximately 7%.4
An essential nutrient in humans, vitamin C is required as a cofactor in the synthesis of mature collagen.3 Collagen is found in skin, bone, and endothelium. Inadequate collagen levels can result in poor dermal support of vessels and tissue fragility, leading to hemorrhage, which can occur in nearly any organ system.
Vitamin C deficiency occurs when serum concentration falls below 11.4
Continue to: Scurvy manifests after 8 to 12 weeks
Scurvy manifests after 8 to 12 weeks of inadequate vitamin C intake.1 Patients may initially experience malaise and irritability. Anemia is common. Dermatologic findings include hyperkeratotic lesions, ecchymoses, poor wound healing, gingival swelling with loss of teeth, petechiae, and corkscrew hairs. Perifollicular hemorrhage is a characteristic finding of scurvy, generally seen on the lower extremities, where the capillaries are under higher hydrostatic pressure.3 Patients may also have musculoskeletal involvement with osteopenia or hemarthroses, which may be seen on imaging.3,5 Cardiorespiratory, gastrointestinal, ophthalmologic, and neurologic findings have also been reported.3
Differential is broad; zero in on patient’s history
The differential diagnosis for hemorrhagic skin lesions is extensive and includes scurvy, coagulopathies, trauma, vasculitis, and vasculopathies.
The presence of perifollicular hemorrhage with corkscrew hairs and a dietary history of inadequate vitamin C intake can differentiate scurvy from other conditions. Serum testing revealing low plasma vitamin C will support the diagnosis, but this is an insensitive test, as values increase with recent intake. Leukocyte ascorbic acid concentrations are more representative of total body stores, but impractical for routine use.6 Skin biopsy is not necessary but may help to rule out other conditions.
Ascorbic acid will facilitate a speedy recovery
Treatment of scurvy includes vitamin C replacement. Response is rapid, with improvement to lethargy within several days and disappearance of other manifestations within several weeks.3 Recommendations on supplementation doses and forms vary, but adults require 300 to 1000 mg/d of ascorbic acid for at least 1 week or until clinical symptoms resolve and stores are repleted.3,5,7
During our patient’s hospital stay, she remained stable and improved clinically with vitamin supplementation (ascorbic acid 1 g/d for 3 days, 500 mg/d after that) and physical therapy. She was counseled on a healthy diet, which would include citrus fruits, tomatoes, and leafy vegetables. The patient was also advised to refrain from drinking alcohol and was given information on an alcohol abstinence program.
At her 1-month follow-up, her condition had improved with near resolution of the skin lesions. She reported that she had given up cigarettes and alcohol. She said she’d also begun eating more citrus fruits and leafy vegetables.
1. Maxfield L, Crane JS. Vitamin C deficiency (scurvy). In: StatPearls. StatPearls Publishing; 2020. Accessed on September 13, 2022. www.ncbi.nlm.nih.gov/books/NBK493187/
2. Worral S. A nightmare disease haunted ships during age of discovery. National Geographic. January 15, 2017. Accessed September 21, 2022. www.nationalgeographic.com/science/article/scurvy-disease-discovery-jonathan-lamb
3. Hirschmann JV, Raugi GJ. Adult Scurvy. J Am Acad Dermatol. 1999;41:895-906. doi: 10.1016/s0190-9622(99)70244-6
4. Schleicher RL, Carroll MD, Ford ES, et al. Serum vitamin C and the prevalence of vitamin C deficiency in the United States: 2003-2004 National Health and Nutrition Examination Survey (NHANES). Am J Clin Nutr. 2009;90:1252-1263. doi: 10.3945/ajcn.2008.27016
5. Agarwal A, Shaharyar A, Kumar A, et al. Scurvy in pediatric age group – A disease often forgotten? J Clin Orthop Trauma. 2015;6:101-107. doi: 10.1016/j.jcot.2014.12.003
6. Scurvy and its prevention and control in major emergencies. World Health Organization. February 23, 1999. Accessed September 13, 2022. www.who.int/publications/i/item/WHO-NHD-99.11
7. Weinstein M, Babyn P, Zlotkin S. An orange a day keeps the doctor away: scurvy in the year 2000. Pediatrics. 2001;108:E55. doi: 10.1542/peds.108.3.e55
A 65-YEAR-OLD WOMAN was brought into the emergency department by her daughter for spontaneous bruising, fatigue, and weakness of several weeks’ duration. She denied taking any medications or illicit drugs and had not experienced any falls or trauma. On a daily basis, she smoked 5 to 7 cigarettes and drank 6 or 7 beers, as had been her custom for several years. The patient lived alone and was grieving the death of her beloved dog, who had died a month earlier. She reported that since the death of her dog, her diet, which hadn’t been especially good to begin with, had deteriorated; it now consisted of beer and crackers.
On admission, she was mildly tachycardic (105 beats/min) with a blood pressure of 125/66 mm Hg. Physical examination revealed a frail-appearing woman who was in no acute distress but was unable to stand without assistance. She had diffuse ecchymoses and perifollicular, purpuric, hyperkeratotic papules and plaques on both of her legs (FIGURES 1A and 1B). In addition, she had faint perifollicular purpuric macules on her upper back. An oral examination revealed poor dentition.
A punch biopsy was performed on her leg, and it revealed noninflammatory dermal hemorrhage without evidence of vasculitis or vasculopathy.
WHAT IS YOUR DIAGNOSIS?
HOW WOULD YOU TREAT THIS PATIENT?
Diagnosis: Scurvy
Based on the patient’s appearance and her dietary history, we suspected scurvy, so a serum vitamin C level was ordered. The results took several days to return. In the meantime, additional lab work revealed hyponatremia (sodium, 129 mmol/L; normal range, 135-145 mmol/L), hypokalemia (potassium, 3 mmol/L; normal range, 3.5-5.2 mmol/L), hypophosphatemia (phosphorus, 2.3 mg/dL; normal range, 2.8-4.5 mg/dL); low serum vitamin D (6 ng/mL; normal range, 20-40 ng/mL); and macrocytic anemia (hemoglobin, 7.4 g/dL; normal range, 11-18 g/dL) with a mean corpuscular volume of 101.1 fL (normal range, 80-100 fL). Her iron panel showed normal serum iron and total iron binding capacity with a normal ferritin level (294 ng/mL; normal range, 30-300 ng/mL). A peripheral blood smear test uncovered mild anisocytosis and polychromasia, with no schistocytes. A fecal immunochemical test was negative.
Several days after admission, the results of the patient’s vitamin C test came back. Her levels were undetectable (< 5 µmol/L; normal range, 11-23 µmol/L), confirming that the patient had scurvy.
A health hazard to marinersthat is still around today
Scurvy is a condition that arises from a deficiency of vitamin C, or ascorbic acid. The first known case of scurvy was in 1550 BC.1 Hippocrates termed the condition “ileos ematitis” and stated that “the mouth feels bad; the gums are detached from the teeth; blood runs from the nostrils … ulcerations on the legs … skin is thin.”1 Scurvy was a major health hazard of mariners between the 15thand 18th centuries.2 Today, the deficiency is uncommon in industrialized countries because there are many sources of vitamin C available through diet and vitamin supplements.3 In the United States, the prevalence of vitamin C deficiency is approximately 7%.4
An essential nutrient in humans, vitamin C is required as a cofactor in the synthesis of mature collagen.3 Collagen is found in skin, bone, and endothelium. Inadequate collagen levels can result in poor dermal support of vessels and tissue fragility, leading to hemorrhage, which can occur in nearly any organ system.
Vitamin C deficiency occurs when serum concentration falls below 11.4
Continue to: Scurvy manifests after 8 to 12 weeks
Scurvy manifests after 8 to 12 weeks of inadequate vitamin C intake.1 Patients may initially experience malaise and irritability. Anemia is common. Dermatologic findings include hyperkeratotic lesions, ecchymoses, poor wound healing, gingival swelling with loss of teeth, petechiae, and corkscrew hairs. Perifollicular hemorrhage is a characteristic finding of scurvy, generally seen on the lower extremities, where the capillaries are under higher hydrostatic pressure.3 Patients may also have musculoskeletal involvement with osteopenia or hemarthroses, which may be seen on imaging.3,5 Cardiorespiratory, gastrointestinal, ophthalmologic, and neurologic findings have also been reported.3
Differential is broad; zero in on patient’s history
The differential diagnosis for hemorrhagic skin lesions is extensive and includes scurvy, coagulopathies, trauma, vasculitis, and vasculopathies.
The presence of perifollicular hemorrhage with corkscrew hairs and a dietary history of inadequate vitamin C intake can differentiate scurvy from other conditions. Serum testing revealing low plasma vitamin C will support the diagnosis, but this is an insensitive test, as values increase with recent intake. Leukocyte ascorbic acid concentrations are more representative of total body stores, but impractical for routine use.6 Skin biopsy is not necessary but may help to rule out other conditions.
Ascorbic acid will facilitate a speedy recovery
Treatment of scurvy includes vitamin C replacement. Response is rapid, with improvement to lethargy within several days and disappearance of other manifestations within several weeks.3 Recommendations on supplementation doses and forms vary, but adults require 300 to 1000 mg/d of ascorbic acid for at least 1 week or until clinical symptoms resolve and stores are repleted.3,5,7
During our patient’s hospital stay, she remained stable and improved clinically with vitamin supplementation (ascorbic acid 1 g/d for 3 days, 500 mg/d after that) and physical therapy. She was counseled on a healthy diet, which would include citrus fruits, tomatoes, and leafy vegetables. The patient was also advised to refrain from drinking alcohol and was given information on an alcohol abstinence program.
At her 1-month follow-up, her condition had improved with near resolution of the skin lesions. She reported that she had given up cigarettes and alcohol. She said she’d also begun eating more citrus fruits and leafy vegetables.
A 65-YEAR-OLD WOMAN was brought into the emergency department by her daughter for spontaneous bruising, fatigue, and weakness of several weeks’ duration. She denied taking any medications or illicit drugs and had not experienced any falls or trauma. On a daily basis, she smoked 5 to 7 cigarettes and drank 6 or 7 beers, as had been her custom for several years. The patient lived alone and was grieving the death of her beloved dog, who had died a month earlier. She reported that since the death of her dog, her diet, which hadn’t been especially good to begin with, had deteriorated; it now consisted of beer and crackers.
On admission, she was mildly tachycardic (105 beats/min) with a blood pressure of 125/66 mm Hg. Physical examination revealed a frail-appearing woman who was in no acute distress but was unable to stand without assistance. She had diffuse ecchymoses and perifollicular, purpuric, hyperkeratotic papules and plaques on both of her legs (FIGURES 1A and 1B). In addition, she had faint perifollicular purpuric macules on her upper back. An oral examination revealed poor dentition.
A punch biopsy was performed on her leg, and it revealed noninflammatory dermal hemorrhage without evidence of vasculitis or vasculopathy.
WHAT IS YOUR DIAGNOSIS?
HOW WOULD YOU TREAT THIS PATIENT?
Diagnosis: Scurvy
Based on the patient’s appearance and her dietary history, we suspected scurvy, so a serum vitamin C level was ordered. The results took several days to return. In the meantime, additional lab work revealed hyponatremia (sodium, 129 mmol/L; normal range, 135-145 mmol/L), hypokalemia (potassium, 3 mmol/L; normal range, 3.5-5.2 mmol/L), hypophosphatemia (phosphorus, 2.3 mg/dL; normal range, 2.8-4.5 mg/dL); low serum vitamin D (6 ng/mL; normal range, 20-40 ng/mL); and macrocytic anemia (hemoglobin, 7.4 g/dL; normal range, 11-18 g/dL) with a mean corpuscular volume of 101.1 fL (normal range, 80-100 fL). Her iron panel showed normal serum iron and total iron binding capacity with a normal ferritin level (294 ng/mL; normal range, 30-300 ng/mL). A peripheral blood smear test uncovered mild anisocytosis and polychromasia, with no schistocytes. A fecal immunochemical test was negative.
Several days after admission, the results of the patient’s vitamin C test came back. Her levels were undetectable (< 5 µmol/L; normal range, 11-23 µmol/L), confirming that the patient had scurvy.
A health hazard to marinersthat is still around today
Scurvy is a condition that arises from a deficiency of vitamin C, or ascorbic acid. The first known case of scurvy was in 1550 BC.1 Hippocrates termed the condition “ileos ematitis” and stated that “the mouth feels bad; the gums are detached from the teeth; blood runs from the nostrils … ulcerations on the legs … skin is thin.”1 Scurvy was a major health hazard of mariners between the 15thand 18th centuries.2 Today, the deficiency is uncommon in industrialized countries because there are many sources of vitamin C available through diet and vitamin supplements.3 In the United States, the prevalence of vitamin C deficiency is approximately 7%.4
An essential nutrient in humans, vitamin C is required as a cofactor in the synthesis of mature collagen.3 Collagen is found in skin, bone, and endothelium. Inadequate collagen levels can result in poor dermal support of vessels and tissue fragility, leading to hemorrhage, which can occur in nearly any organ system.
Vitamin C deficiency occurs when serum concentration falls below 11.4
Continue to: Scurvy manifests after 8 to 12 weeks
Scurvy manifests after 8 to 12 weeks of inadequate vitamin C intake.1 Patients may initially experience malaise and irritability. Anemia is common. Dermatologic findings include hyperkeratotic lesions, ecchymoses, poor wound healing, gingival swelling with loss of teeth, petechiae, and corkscrew hairs. Perifollicular hemorrhage is a characteristic finding of scurvy, generally seen on the lower extremities, where the capillaries are under higher hydrostatic pressure.3 Patients may also have musculoskeletal involvement with osteopenia or hemarthroses, which may be seen on imaging.3,5 Cardiorespiratory, gastrointestinal, ophthalmologic, and neurologic findings have also been reported.3
Differential is broad; zero in on patient’s history
The differential diagnosis for hemorrhagic skin lesions is extensive and includes scurvy, coagulopathies, trauma, vasculitis, and vasculopathies.
The presence of perifollicular hemorrhage with corkscrew hairs and a dietary history of inadequate vitamin C intake can differentiate scurvy from other conditions. Serum testing revealing low plasma vitamin C will support the diagnosis, but this is an insensitive test, as values increase with recent intake. Leukocyte ascorbic acid concentrations are more representative of total body stores, but impractical for routine use.6 Skin biopsy is not necessary but may help to rule out other conditions.
Ascorbic acid will facilitate a speedy recovery
Treatment of scurvy includes vitamin C replacement. Response is rapid, with improvement to lethargy within several days and disappearance of other manifestations within several weeks.3 Recommendations on supplementation doses and forms vary, but adults require 300 to 1000 mg/d of ascorbic acid for at least 1 week or until clinical symptoms resolve and stores are repleted.3,5,7
During our patient’s hospital stay, she remained stable and improved clinically with vitamin supplementation (ascorbic acid 1 g/d for 3 days, 500 mg/d after that) and physical therapy. She was counseled on a healthy diet, which would include citrus fruits, tomatoes, and leafy vegetables. The patient was also advised to refrain from drinking alcohol and was given information on an alcohol abstinence program.
At her 1-month follow-up, her condition had improved with near resolution of the skin lesions. She reported that she had given up cigarettes and alcohol. She said she’d also begun eating more citrus fruits and leafy vegetables.
1. Maxfield L, Crane JS. Vitamin C deficiency (scurvy). In: StatPearls. StatPearls Publishing; 2020. Accessed on September 13, 2022. www.ncbi.nlm.nih.gov/books/NBK493187/
2. Worral S. A nightmare disease haunted ships during age of discovery. National Geographic. January 15, 2017. Accessed September 21, 2022. www.nationalgeographic.com/science/article/scurvy-disease-discovery-jonathan-lamb
3. Hirschmann JV, Raugi GJ. Adult Scurvy. J Am Acad Dermatol. 1999;41:895-906. doi: 10.1016/s0190-9622(99)70244-6
4. Schleicher RL, Carroll MD, Ford ES, et al. Serum vitamin C and the prevalence of vitamin C deficiency in the United States: 2003-2004 National Health and Nutrition Examination Survey (NHANES). Am J Clin Nutr. 2009;90:1252-1263. doi: 10.3945/ajcn.2008.27016
5. Agarwal A, Shaharyar A, Kumar A, et al. Scurvy in pediatric age group – A disease often forgotten? J Clin Orthop Trauma. 2015;6:101-107. doi: 10.1016/j.jcot.2014.12.003
6. Scurvy and its prevention and control in major emergencies. World Health Organization. February 23, 1999. Accessed September 13, 2022. www.who.int/publications/i/item/WHO-NHD-99.11
7. Weinstein M, Babyn P, Zlotkin S. An orange a day keeps the doctor away: scurvy in the year 2000. Pediatrics. 2001;108:E55. doi: 10.1542/peds.108.3.e55
1. Maxfield L, Crane JS. Vitamin C deficiency (scurvy). In: StatPearls. StatPearls Publishing; 2020. Accessed on September 13, 2022. www.ncbi.nlm.nih.gov/books/NBK493187/
2. Worral S. A nightmare disease haunted ships during age of discovery. National Geographic. January 15, 2017. Accessed September 21, 2022. www.nationalgeographic.com/science/article/scurvy-disease-discovery-jonathan-lamb
3. Hirschmann JV, Raugi GJ. Adult Scurvy. J Am Acad Dermatol. 1999;41:895-906. doi: 10.1016/s0190-9622(99)70244-6
4. Schleicher RL, Carroll MD, Ford ES, et al. Serum vitamin C and the prevalence of vitamin C deficiency in the United States: 2003-2004 National Health and Nutrition Examination Survey (NHANES). Am J Clin Nutr. 2009;90:1252-1263. doi: 10.3945/ajcn.2008.27016
5. Agarwal A, Shaharyar A, Kumar A, et al. Scurvy in pediatric age group – A disease often forgotten? J Clin Orthop Trauma. 2015;6:101-107. doi: 10.1016/j.jcot.2014.12.003
6. Scurvy and its prevention and control in major emergencies. World Health Organization. February 23, 1999. Accessed September 13, 2022. www.who.int/publications/i/item/WHO-NHD-99.11
7. Weinstein M, Babyn P, Zlotkin S. An orange a day keeps the doctor away: scurvy in the year 2000. Pediatrics. 2001;108:E55. doi: 10.1542/peds.108.3.e55
Does an early COPD diagnosis improve long-term outcomes?
EVIDENCE SUMMARY
Early Dx didn’t improve smoking cessation rates or treatment outcomes
A 2016 evidence report and systematic review for the US Preventive Services Task Force (USPSTF) identified no studies directly comparing the effectiveness of COPD screening on patient outcomes, so the authors looked first at studies on the outcomes of screening, followed by studies exploring the effects of early treatment.1
The authors identified 5 fair-quality RCTs (N = 1694) addressing the effect of screening asymptomatic patients for COPD with spirometry on the outcome of smoking cessation. One trial (n = 561) found better 12-month smoking cessation rates in patients who underwent spirometry screening and were given their “lung age” (13.6% vs 6.4% not given a lung age; P < .005; number needed to treat [NNT] = 14). However, a similar study (n = 542) published a year later found no significant difference in quit rates with or without “lung age” discussions (10.9% vs 13%, respectively; P not significant). In the other 3 studies, screening produced no significant effect on smoking cessation rates.1
As for possible early treatment benefits, the review authors identified only 1 RCT (n = 1175) that included any patients with mild COPD (defined as COPD with a forced expiratory volume in 1 second [FEV1] ≥ 80% of predicted normal value). It assessed treatment with inhaled corticosteroids (ICS) in patients with mild COPD who continued to smoke. The trial did not record symptoms (if any) at intake. ICS therapy reduced the frequency of COPD exacerbations (relative risk = 0.63; 95% CI, 0.47-0.85), although patients with milder COPD benefitted little in absolute terms (by 0.02 exacerbations/year).1 The review authors further noted that data were insufficient to make definitive statements about the effect of ICS on dyspnea or health-related quality of life.
But later diagnosis is associated with poorer outcomes
Two recent, large retrospective observational cohort studies, however, have examined the impact of an early vs late COPD diagnosis in patients with dyspnea or other symptoms of COPD.2,3 A later diagnosis was associated with worse outcomes.
In the first study, researchers in Sweden identified patients older than 40 years who had received a new diagnosis of COPD between 2000 and 2014.2 They examined electronic health record data for 6 different “indicators” of COPD during the 5 years prior to date of diagnosis: pneumonia, other respiratory disease, oral steroids, antibiotics for respiratory infection, prescribed drugs for respiratory symptoms, and lung function measurement. Researchers categorized patients as early diagnosis (if they had ≤ 2 indicators prior to diagnosis) or late diagnosis (≥ 3 indicators prior to diagnosis). Compared with early diagnosis (n = 3870), late diagnosis (n = 8827) was associated with
- a higher annual rate of exacerbations within the first 2 years after diagnosis (2.67 vs 1.41; hazard ratio [HR] = 1.89; 95% CI, 1.83-1.96; P < .0001; number of early diagnoses needed to prevent 1 exacerbation in 1 year = 79),
- shorter time to first exacerbation (HR = 1.61; 95% CI, 1.54-1.69; P < .0001), and
- higher direct health care costs (by €1500 per year; no P value given).
Mortality was not different between the groups (HR = 1.04; 95% CI, 0.98-1.11; P = .18).
The second investigation was a similarly designed retrospective observational cohort study using a large UK database.3 Researchers enrolled patients who were at least 40 years old and received a new diagnosis of COPD between 2011 and 2014.
Continue to: Researchers examined electronic...
Researchers examined electronic health record data in the 5 years prior to diagnosis for 7 possible indicators of early COPD: pneumonia, respiratory disease other than pneumonia, chest radiograph, prescription of oral steroids, prescription of antibiotics for lung infection, prescription to manage respiratory disease symptoms, and lung function measurement. Researchers categorized patients as early diagnosis (≥ 2 indicators prior to diagnosis) or late diagnosis (≥ 3 indicators prior to diagnosis). Compared with early diagnosis (n = 3375), late diagnosis (n = 6783) was associated with a higher annual rate of exacerbations over 3-year follow-up (1.09 vs 0.57; adjusted HR = 1.68; 95% CI, 1.59-1.79; P < .0001; or 1 additional exacerbation in 192 patients in 1 year), shorter mean time to first exacerbation (HR = 1.46; 95% CI: 1.38-1.55; P < .0001), and a higher risk of hospitalization within 3 years (rate ratio = 1.18; 95% CI, 1.08-1.28; P = .0001). The researchers did not evaluate for mortality.
Importantly, patients in the late COPD diagnosis group in both trials had higher rates of other severe illnesses that cause dyspnea, including cardiovascular disease and other pulmonary diseases. As a result, dyspnea of other etiologies may have contributed to both the later diagnoses and the poorer clinical outcomes of the late-diagnosis group. Both studies had a high risk of lead-time bias.
Recommendations from others
In 2016, the USPSTF gave a “D” rating (moderate or high certainty that the service has no net benefit or that the harms outweigh the benefits) to screening asymptomatic adults without respiratory symptoms for COPD.4 Likewise, the 2017 Global Initiative for Chronic Obstructive Lung Disease (GOLD) report did not recommend routine screening with spirometry but did advocate trying to make an accurate diagnosis using spirometry in patients with risk factors for COPD and chronic, progressive symptoms.5
Editor’s takeaway
Reasonably good evidence failed to find a benefit from an early COPD diagnosis. Even smoking cessation rates were not improved. Without better disease-modifying treatments, spirometry—the gold standard for confirming a COPD diagnosis—should not be used for screening asymptomatic patients.
1. Guirguis-Blake JM, Senger CA, Webber EM, et al. Screening for chronic obstructive pulmonary disease: evidence report and systematic review for the US Preventive Services Task Force. JAMA. 2016;315:1378-1393. doi:10.1001/jama.2016.2654
2. Larsson K, Janson C, Ställberg B, et al. Impact of COPD diagnosis timing on clinical and economic outcomes: the ARCTIC observational cohort study. Int J Chron Obstruct Pulmon Dis. 2019;14:995-1008. doi: 10.2147/COPD.S195382
3. Kostikas K, Price D, Gutzwiller FS, et al. Clinical impact and healthcare resource utilization associated with early versus late COPD diagnosis in patients from UK CPRD database. Int J Chron Obstruct Pulmon Dis. 2020;15:1729-1738. doi: 10.2147/COPD.S255414
4. US Preventive Services Task Force; Siu AL, Bibbins-Domingo K, Grossman DC, et al. Screening for chronic obstructive pulmonary disease: US Preventive Services Task Force recommendation statement. JAMA. 2016;315:1372-1377. doi: 10.1001/jama.2016.2638
5. Vogelmeier CF, Criner GJ, Martinez FJ, et al. Global strategy for the diagnosis, management, and prevention of chronic obstructive lung disease 2017 report. GOLD executive summary. Am J Respir Crit Care Med. 2017;195:557-582. doi: 10.1164/rccm.201701-0218PP
EVIDENCE SUMMARY
Early Dx didn’t improve smoking cessation rates or treatment outcomes
A 2016 evidence report and systematic review for the US Preventive Services Task Force (USPSTF) identified no studies directly comparing the effectiveness of COPD screening on patient outcomes, so the authors looked first at studies on the outcomes of screening, followed by studies exploring the effects of early treatment.1
The authors identified 5 fair-quality RCTs (N = 1694) addressing the effect of screening asymptomatic patients for COPD with spirometry on the outcome of smoking cessation. One trial (n = 561) found better 12-month smoking cessation rates in patients who underwent spirometry screening and were given their “lung age” (13.6% vs 6.4% not given a lung age; P < .005; number needed to treat [NNT] = 14). However, a similar study (n = 542) published a year later found no significant difference in quit rates with or without “lung age” discussions (10.9% vs 13%, respectively; P not significant). In the other 3 studies, screening produced no significant effect on smoking cessation rates.1
As for possible early treatment benefits, the review authors identified only 1 RCT (n = 1175) that included any patients with mild COPD (defined as COPD with a forced expiratory volume in 1 second [FEV1] ≥ 80% of predicted normal value). It assessed treatment with inhaled corticosteroids (ICS) in patients with mild COPD who continued to smoke. The trial did not record symptoms (if any) at intake. ICS therapy reduced the frequency of COPD exacerbations (relative risk = 0.63; 95% CI, 0.47-0.85), although patients with milder COPD benefitted little in absolute terms (by 0.02 exacerbations/year).1 The review authors further noted that data were insufficient to make definitive statements about the effect of ICS on dyspnea or health-related quality of life.
But later diagnosis is associated with poorer outcomes
Two recent, large retrospective observational cohort studies, however, have examined the impact of an early vs late COPD diagnosis in patients with dyspnea or other symptoms of COPD.2,3 A later diagnosis was associated with worse outcomes.
In the first study, researchers in Sweden identified patients older than 40 years who had received a new diagnosis of COPD between 2000 and 2014.2 They examined electronic health record data for 6 different “indicators” of COPD during the 5 years prior to date of diagnosis: pneumonia, other respiratory disease, oral steroids, antibiotics for respiratory infection, prescribed drugs for respiratory symptoms, and lung function measurement. Researchers categorized patients as early diagnosis (if they had ≤ 2 indicators prior to diagnosis) or late diagnosis (≥ 3 indicators prior to diagnosis). Compared with early diagnosis (n = 3870), late diagnosis (n = 8827) was associated with
- a higher annual rate of exacerbations within the first 2 years after diagnosis (2.67 vs 1.41; hazard ratio [HR] = 1.89; 95% CI, 1.83-1.96; P < .0001; number of early diagnoses needed to prevent 1 exacerbation in 1 year = 79),
- shorter time to first exacerbation (HR = 1.61; 95% CI, 1.54-1.69; P < .0001), and
- higher direct health care costs (by €1500 per year; no P value given).
Mortality was not different between the groups (HR = 1.04; 95% CI, 0.98-1.11; P = .18).
The second investigation was a similarly designed retrospective observational cohort study using a large UK database.3 Researchers enrolled patients who were at least 40 years old and received a new diagnosis of COPD between 2011 and 2014.
Continue to: Researchers examined electronic...
Researchers examined electronic health record data in the 5 years prior to diagnosis for 7 possible indicators of early COPD: pneumonia, respiratory disease other than pneumonia, chest radiograph, prescription of oral steroids, prescription of antibiotics for lung infection, prescription to manage respiratory disease symptoms, and lung function measurement. Researchers categorized patients as early diagnosis (≥ 2 indicators prior to diagnosis) or late diagnosis (≥ 3 indicators prior to diagnosis). Compared with early diagnosis (n = 3375), late diagnosis (n = 6783) was associated with a higher annual rate of exacerbations over 3-year follow-up (1.09 vs 0.57; adjusted HR = 1.68; 95% CI, 1.59-1.79; P < .0001; or 1 additional exacerbation in 192 patients in 1 year), shorter mean time to first exacerbation (HR = 1.46; 95% CI: 1.38-1.55; P < .0001), and a higher risk of hospitalization within 3 years (rate ratio = 1.18; 95% CI, 1.08-1.28; P = .0001). The researchers did not evaluate for mortality.
Importantly, patients in the late COPD diagnosis group in both trials had higher rates of other severe illnesses that cause dyspnea, including cardiovascular disease and other pulmonary diseases. As a result, dyspnea of other etiologies may have contributed to both the later diagnoses and the poorer clinical outcomes of the late-diagnosis group. Both studies had a high risk of lead-time bias.
Recommendations from others
In 2016, the USPSTF gave a “D” rating (moderate or high certainty that the service has no net benefit or that the harms outweigh the benefits) to screening asymptomatic adults without respiratory symptoms for COPD.4 Likewise, the 2017 Global Initiative for Chronic Obstructive Lung Disease (GOLD) report did not recommend routine screening with spirometry but did advocate trying to make an accurate diagnosis using spirometry in patients with risk factors for COPD and chronic, progressive symptoms.5
Editor’s takeaway
Reasonably good evidence failed to find a benefit from an early COPD diagnosis. Even smoking cessation rates were not improved. Without better disease-modifying treatments, spirometry—the gold standard for confirming a COPD diagnosis—should not be used for screening asymptomatic patients.
EVIDENCE SUMMARY
Early Dx didn’t improve smoking cessation rates or treatment outcomes
A 2016 evidence report and systematic review for the US Preventive Services Task Force (USPSTF) identified no studies directly comparing the effectiveness of COPD screening on patient outcomes, so the authors looked first at studies on the outcomes of screening, followed by studies exploring the effects of early treatment.1
The authors identified 5 fair-quality RCTs (N = 1694) addressing the effect of screening asymptomatic patients for COPD with spirometry on the outcome of smoking cessation. One trial (n = 561) found better 12-month smoking cessation rates in patients who underwent spirometry screening and were given their “lung age” (13.6% vs 6.4% not given a lung age; P < .005; number needed to treat [NNT] = 14). However, a similar study (n = 542) published a year later found no significant difference in quit rates with or without “lung age” discussions (10.9% vs 13%, respectively; P not significant). In the other 3 studies, screening produced no significant effect on smoking cessation rates.1
As for possible early treatment benefits, the review authors identified only 1 RCT (n = 1175) that included any patients with mild COPD (defined as COPD with a forced expiratory volume in 1 second [FEV1] ≥ 80% of predicted normal value). It assessed treatment with inhaled corticosteroids (ICS) in patients with mild COPD who continued to smoke. The trial did not record symptoms (if any) at intake. ICS therapy reduced the frequency of COPD exacerbations (relative risk = 0.63; 95% CI, 0.47-0.85), although patients with milder COPD benefitted little in absolute terms (by 0.02 exacerbations/year).1 The review authors further noted that data were insufficient to make definitive statements about the effect of ICS on dyspnea or health-related quality of life.
But later diagnosis is associated with poorer outcomes
Two recent, large retrospective observational cohort studies, however, have examined the impact of an early vs late COPD diagnosis in patients with dyspnea or other symptoms of COPD.2,3 A later diagnosis was associated with worse outcomes.
In the first study, researchers in Sweden identified patients older than 40 years who had received a new diagnosis of COPD between 2000 and 2014.2 They examined electronic health record data for 6 different “indicators” of COPD during the 5 years prior to date of diagnosis: pneumonia, other respiratory disease, oral steroids, antibiotics for respiratory infection, prescribed drugs for respiratory symptoms, and lung function measurement. Researchers categorized patients as early diagnosis (if they had ≤ 2 indicators prior to diagnosis) or late diagnosis (≥ 3 indicators prior to diagnosis). Compared with early diagnosis (n = 3870), late diagnosis (n = 8827) was associated with
- a higher annual rate of exacerbations within the first 2 years after diagnosis (2.67 vs 1.41; hazard ratio [HR] = 1.89; 95% CI, 1.83-1.96; P < .0001; number of early diagnoses needed to prevent 1 exacerbation in 1 year = 79),
- shorter time to first exacerbation (HR = 1.61; 95% CI, 1.54-1.69; P < .0001), and
- higher direct health care costs (by €1500 per year; no P value given).
Mortality was not different between the groups (HR = 1.04; 95% CI, 0.98-1.11; P = .18).
The second investigation was a similarly designed retrospective observational cohort study using a large UK database.3 Researchers enrolled patients who were at least 40 years old and received a new diagnosis of COPD between 2011 and 2014.
Continue to: Researchers examined electronic...
Researchers examined electronic health record data in the 5 years prior to diagnosis for 7 possible indicators of early COPD: pneumonia, respiratory disease other than pneumonia, chest radiograph, prescription of oral steroids, prescription of antibiotics for lung infection, prescription to manage respiratory disease symptoms, and lung function measurement. Researchers categorized patients as early diagnosis (≥ 2 indicators prior to diagnosis) or late diagnosis (≥ 3 indicators prior to diagnosis). Compared with early diagnosis (n = 3375), late diagnosis (n = 6783) was associated with a higher annual rate of exacerbations over 3-year follow-up (1.09 vs 0.57; adjusted HR = 1.68; 95% CI, 1.59-1.79; P < .0001; or 1 additional exacerbation in 192 patients in 1 year), shorter mean time to first exacerbation (HR = 1.46; 95% CI: 1.38-1.55; P < .0001), and a higher risk of hospitalization within 3 years (rate ratio = 1.18; 95% CI, 1.08-1.28; P = .0001). The researchers did not evaluate for mortality.
Importantly, patients in the late COPD diagnosis group in both trials had higher rates of other severe illnesses that cause dyspnea, including cardiovascular disease and other pulmonary diseases. As a result, dyspnea of other etiologies may have contributed to both the later diagnoses and the poorer clinical outcomes of the late-diagnosis group. Both studies had a high risk of lead-time bias.
Recommendations from others
In 2016, the USPSTF gave a “D” rating (moderate or high certainty that the service has no net benefit or that the harms outweigh the benefits) to screening asymptomatic adults without respiratory symptoms for COPD.4 Likewise, the 2017 Global Initiative for Chronic Obstructive Lung Disease (GOLD) report did not recommend routine screening with spirometry but did advocate trying to make an accurate diagnosis using spirometry in patients with risk factors for COPD and chronic, progressive symptoms.5
Editor’s takeaway
Reasonably good evidence failed to find a benefit from an early COPD diagnosis. Even smoking cessation rates were not improved. Without better disease-modifying treatments, spirometry—the gold standard for confirming a COPD diagnosis—should not be used for screening asymptomatic patients.
1. Guirguis-Blake JM, Senger CA, Webber EM, et al. Screening for chronic obstructive pulmonary disease: evidence report and systematic review for the US Preventive Services Task Force. JAMA. 2016;315:1378-1393. doi:10.1001/jama.2016.2654
2. Larsson K, Janson C, Ställberg B, et al. Impact of COPD diagnosis timing on clinical and economic outcomes: the ARCTIC observational cohort study. Int J Chron Obstruct Pulmon Dis. 2019;14:995-1008. doi: 10.2147/COPD.S195382
3. Kostikas K, Price D, Gutzwiller FS, et al. Clinical impact and healthcare resource utilization associated with early versus late COPD diagnosis in patients from UK CPRD database. Int J Chron Obstruct Pulmon Dis. 2020;15:1729-1738. doi: 10.2147/COPD.S255414
4. US Preventive Services Task Force; Siu AL, Bibbins-Domingo K, Grossman DC, et al. Screening for chronic obstructive pulmonary disease: US Preventive Services Task Force recommendation statement. JAMA. 2016;315:1372-1377. doi: 10.1001/jama.2016.2638
5. Vogelmeier CF, Criner GJ, Martinez FJ, et al. Global strategy for the diagnosis, management, and prevention of chronic obstructive lung disease 2017 report. GOLD executive summary. Am J Respir Crit Care Med. 2017;195:557-582. doi: 10.1164/rccm.201701-0218PP
1. Guirguis-Blake JM, Senger CA, Webber EM, et al. Screening for chronic obstructive pulmonary disease: evidence report and systematic review for the US Preventive Services Task Force. JAMA. 2016;315:1378-1393. doi:10.1001/jama.2016.2654
2. Larsson K, Janson C, Ställberg B, et al. Impact of COPD diagnosis timing on clinical and economic outcomes: the ARCTIC observational cohort study. Int J Chron Obstruct Pulmon Dis. 2019;14:995-1008. doi: 10.2147/COPD.S195382
3. Kostikas K, Price D, Gutzwiller FS, et al. Clinical impact and healthcare resource utilization associated with early versus late COPD diagnosis in patients from UK CPRD database. Int J Chron Obstruct Pulmon Dis. 2020;15:1729-1738. doi: 10.2147/COPD.S255414
4. US Preventive Services Task Force; Siu AL, Bibbins-Domingo K, Grossman DC, et al. Screening for chronic obstructive pulmonary disease: US Preventive Services Task Force recommendation statement. JAMA. 2016;315:1372-1377. doi: 10.1001/jama.2016.2638
5. Vogelmeier CF, Criner GJ, Martinez FJ, et al. Global strategy for the diagnosis, management, and prevention of chronic obstructive lung disease 2017 report. GOLD executive summary. Am J Respir Crit Care Med. 2017;195:557-582. doi: 10.1164/rccm.201701-0218PP
EVIDENCE-BASED ANSWER:
It depends. A diagnosis of chronic obstructive pulmonary disease (COPD) made using screening spirometry in patients without symptoms does not change the course of the disease or alter smoking rates (strength of recommendation [SOR]: A, preponderance of evidence from multiple randomized controlled trials [RCTs]). However, once a patient develops symptoms of lung disease, a delayed diagnosis is associated with poorer outcomes (SOR: B, cohort studies). Active case finding (including the use of spirometry) is recommended for patients with risk factors for COPD who present with consistent symptoms (SOR: C, expert opinion).
A “no-biopsy” approach to diagnosing celiac disease
ILLUSTRATIVE CASE
A 43-year-old woman presents to the clinic with diffuse, intermittent abdominal discomfort, bloating, and diarrhea that has slowly but steadily worsened over the past few years to now-daily symptoms. She states her overall health is otherwise good. Her review of systems is pertinent only for 8 lbs of unintentional weight loss over the past year and increased fatigue. She takes no supplements or routine over-the-counter or prescription medications, except for low-dose combination oral contraceptives, and is unaware of any family history of gastrointestinal (GI) diseases. She does not drink or smoke. She is up to date with immunizations and with cervical and breast cancer screening. Her body mass index is 23, her vital signs are within normal limits, and her physical exam is normal except for mild, diffuse abdominal tenderness without any masses, organomegaly, or peritoneal signs.
Her diagnostic work-up includes a complete metabolic panel, magnesium level, complete blood count, thyroid-stimulating hormone measurement, cytomegalovirus IgG and IgM serology, and stool studies for fecal leukocytes, ova and parasites, and fecal fat, in addition to a kidney, ureter, and bladder noncontrast computed tomography scan. All diagnostic testing is negative except for slightly elevated fecal fat, thereby decreasing the likelihood of infection, thyroid disorder, electrolyte abnormalities, or malignancy as a source of her symptoms.
She says that based on her online searches, her symptoms seem consistent with CD—with which you concur. However, she is fearful of an endoscopic procedure and asks if there is any other way to diagnose CD.
CD is an immune-mediated disorder in genetically susceptible people that is triggered by dietary gluten, causing damage to the small intestine.1-6 The estimated worldwide prevalence of CD is approximately 1%, with greater prevalence in females.1-6 A strong genetic predisposition also has been noted: prevalence among first-degree relatives is 10% to 44%.2,3,6 Although CD can be diagnosed at any age, in the United States the mean age at diagnosis is in the fifth decade of life.6
The incidence of CD is on the rise due to true increases in disease incidence and prevalence, increased detection through better diagnostic tools, and increased screening of at-risk populations (eg, first-degree relatives, those with specific human leukocyte antigen variant genotypes, and those with certain chromosomal disorders, such as Down syndrome and Turner syndrome).2-6 However, despite the increasing prevalence of CD, most patients remain undiagnosed.1
The diagnosis of CD in adults is typically made with elevated serum tTG-IgA and
STUDY SUMMARY
tTG-IgA titers were highly predictive of CD in 3 distinct cohorts
This 2021 hybrid prospective/retrospective study with 3 distinct cohorts aimed to assess the utility of serum tTG-IgA titers compared to traditional EGD with duodenal biopsy for the diagnosis of CD in adult participants (defined as ≥ 16 years of age). A serum tTG-IgA titer ≥ 10 times the ULN was set as the minimal cutoff value, and standardized duodenal biopsy sampling and evaluation for histologic mucosal changes consistent with
Continue to: Cohort 1 was a...
Cohort 1 was a prospective analysis of adults (N = 740) considered to have a high suspicion for CD, recruited from a single CD subspecialty clinic in the United Kingdom. Patients with a previous diagnosis of CD, those adhering to a gluten-free diet, and those with IgA deficiency were excluded. Study patients had tTG-IgA titers drawn and, within 6 weeks, underwent endoscopy with ≥ 1 biopsy from the duodenal bulb and/or the second part of the duodenum. The PPV of tTG-IgA titers ≥ 10 times the ULN in patients with biopsy-proven CD was 98.7% (95% CI, 97%-99.4%).
Cohort 2 was a retrospective analysis of adult patients (N = 532) considered to have low suspicion for CD. These patients were referred for endoscopy for generalized GI complaints in the same hospital as Cohort 1, but not the subspecialty clinic. Exclusion criteria and timing of IgA titers and endoscopy were identical to those of Cohort 1. The PPV of tTG-IgA titers ≥ 10 times the ULN in patients with biopsy-proven CD was 100%.
Cohort 3 (which included patients in 8 countries) was a retrospective analysis of the performance of multiple assays to enhance the validity of this approach in a wide range of settings. Adult patients (N = 145) with tTG-IgA serology positive for celiac who then underwent endoscopy with 4 to 6 duodenal biopsy samples were included in this analysis. Eleven distinct laboratories performed the tTG-IgA assay. The PPV of tTG-IgA titers ≥ 10 times the ULN in patients with biopsy-proven CD was 95.2% (95% CI, 84.6%-98.6%).
In total, this study included 1417 adult patients; 431 (30%) had tTG-IgA titers ≥ 10 times the ULN. Of those patients, 424 (98%) had histopathologic findings on duodenal biopsy consistent with CD.
Of note, there was no standardization as to the assays used for the tTG-IgA titers: Cohort 1 used 2 different manufacturers’ assays, Cohort 2 used 1 assay, and Cohort 3 used 5 assays. Regardless, the “≥ 10 times the ULN” calculation was based on each manufacturer’s published assay ranges. The lack of assay standardization did create variance in false-positive rates, however: Across all 3 cohorts, the false-positive rate for trusting the “≥ 10 times the ULN” threshold as the sole marker for CD in adults increased from 1% (Cohorts 1 and 2) to 5% (all 3 cohorts).
Continue to: WHAT'S NEW
WHAT’S NEW
Less invasive, less costly diagnosis of celiac disease in adults
In adults with symptoms suggestive of CD, the diagnosis can be made with a high level of certainty if a serum tTG-IgA titer is ≥ 10 times the ULN. Through informed, shared decision making in the presence of such a finding, patients may accept a serologic diagnosis and forgo an invasive EGD with biopsy and its inherent costs and risks. Indeed, if the majority of patients with CD are undiagnosed or underdiagnosed, and there exists a minimally invasive blood test that is highly cost effective in the absence of “red flags,” the overall benefit of this path could be substantial.
CAVEATS
“No biopsy” does not mean no risk/benefit discussion
While the PPVs are quite high, the negative predictive value varied greatly: 13%, 98%, and 10% for Cohorts 1, 2, and 3, respectively. Therefore, although serum tTG-IgA titers ≥ 10 times the ULN are useful for diagnosis, a negative result (serum tTG-IgA titers < 10 times the ULN) should not be used to rule out CD, and other testing should be pursued.
Additionally (although rare), patients with CD who have IgA deficiency may obtain false-negative results using the tTG-IgA ≥ 10 times the ULN diagnostic criterion.7,8
Also, both Cohorts 1 and 2 took place in general or subspecialty GI clinics (Cohort 3’s site types were not specified). However, the objective interpretation of tTG-IgA serology means it could be considered as an additional diagnostic tool for primary care physicians, as well.
Finally, if a primary care physician and their patient decide to go the “no-biopsy” route, it should be with a full discussion of the possible risks and benefits of not pursuing EGD. If there are any potential “red flag” symptoms suggesting the possibility of a more concerning differential diagnosis, EGD evaluation should still be pursued. Such symptoms might include (but not be limited to) chronic dyspepsia, dysphagia, weight loss, and unexplained anemia.7
Continue to: CHALLENGES TO IMPLEMENTATION
CHALLENGES TO IMPLEMENTATION
Diagnostic guidelines still favor EGD with biopsy for adults
The 2013 American College of Gastroenterology guidelines support the use of EGD and duodenal biopsy to diagnose CD in both low- and high-risk patients, regardless of serologic findings.7 In a 2019 Clinical Practice Update, the American Gastrointestinal Association (AGA) stated that when tTG-IgA titers are ≥ 10 times the ULN and EMAs are positive, the PPV is “virtually 100%” for CD. Yet they still state that in this scenario “EGD and duodenal biopsies may then be performed for purposes of differential diagnosis.”8 Furthermore, the AGA does not discuss informed and shared decision making with patients for the option of a “no-biopsy” diagnosis.8
Additionally, there may be challenges in finding commercial laboratories that report reference ranges with a clear ULN. Although costs for the serum tTG-IgA assay vary, they are less expensive than endoscopy with biopsy and histopathologic examination, and therefore may present less of a financial barrier.
1. Penny HA, Raju SA, Lau MS, et al. Accuracy of a no-biopsy approach for the diagnosis of coeliac disease across different adult cohorts. Gut. 2021;70:876-883. doi: 10.1136/gutjnl-2020-320913
2. Al-Toma A, Volta U, Auricchio R, et al. European Society for the Study of Coeliac Disease (ESsCD) guideline for coeliac disease and other gluten-related disorders. United European Gastroenterol J. 2019;7:583-613. doi: 10.1177/2050640619844125
3. Caio G, Volta U, Sapone A, et al. Celiac disease: a comprehensive current review. BMC Med. 2019;17:142. doi: 10.1186/s12916-019-1380-z
4. Lebwohl B, Rubio-Tapia A. Epidemiology, presentation, and diagnosis of celiac disease. Gastroenterology. 2021;160:63-75. doi: 10.1053/j.gastro.2020.06.098
5. Lebwohl B, Sanders DS, Green PHR. Coeliac disease. Lancet. 2018;391:70-81. doi: 10.1016/S0140-6736(17)31796-8
6. Rubin JE, Crowe SE. Celiac disease. Ann Intern Med. 2020;172:ITC1-ITC16. doi: 10.7326/AITC202001070
7. Rubio-Tapia A, Hill ID, Kelly CP, et al; American College of Gastroenterology. ACG clinical guidelines: diagnosis and management of celiac disease. Am J Gastroenterol. 2013;108:656-676; quiz 677. doi: 10.1038/ajg.2013.79
8. Husby S, Murray JA, Katzka DA. AGA clinical practice update on diagnosis and monitoring of celiac disease—changing utility of serology and histologic measures: expert review. Gastroenterology. 2019;156:885-889. doi: 10.1053/j.gastro.2018.12.010
9. Husby S, Koletzko S, Korponay-Szabó I, et al. European Society Paediatric Gastroenterology, Hepatology and Nutrition guidelines for diagnosing coeliac disease 2020. J Pediatr Gastroenterol Nutr. 2020;70:141-156. doi: 10.1097/MPG.0000000000002497
ILLUSTRATIVE CASE
A 43-year-old woman presents to the clinic with diffuse, intermittent abdominal discomfort, bloating, and diarrhea that has slowly but steadily worsened over the past few years to now-daily symptoms. She states her overall health is otherwise good. Her review of systems is pertinent only for 8 lbs of unintentional weight loss over the past year and increased fatigue. She takes no supplements or routine over-the-counter or prescription medications, except for low-dose combination oral contraceptives, and is unaware of any family history of gastrointestinal (GI) diseases. She does not drink or smoke. She is up to date with immunizations and with cervical and breast cancer screening. Her body mass index is 23, her vital signs are within normal limits, and her physical exam is normal except for mild, diffuse abdominal tenderness without any masses, organomegaly, or peritoneal signs.
Her diagnostic work-up includes a complete metabolic panel, magnesium level, complete blood count, thyroid-stimulating hormone measurement, cytomegalovirus IgG and IgM serology, and stool studies for fecal leukocytes, ova and parasites, and fecal fat, in addition to a kidney, ureter, and bladder noncontrast computed tomography scan. All diagnostic testing is negative except for slightly elevated fecal fat, thereby decreasing the likelihood of infection, thyroid disorder, electrolyte abnormalities, or malignancy as a source of her symptoms.
She says that based on her online searches, her symptoms seem consistent with CD—with which you concur. However, she is fearful of an endoscopic procedure and asks if there is any other way to diagnose CD.
CD is an immune-mediated disorder in genetically susceptible people that is triggered by dietary gluten, causing damage to the small intestine.1-6 The estimated worldwide prevalence of CD is approximately 1%, with greater prevalence in females.1-6 A strong genetic predisposition also has been noted: prevalence among first-degree relatives is 10% to 44%.2,3,6 Although CD can be diagnosed at any age, in the United States the mean age at diagnosis is in the fifth decade of life.6
The incidence of CD is on the rise due to true increases in disease incidence and prevalence, increased detection through better diagnostic tools, and increased screening of at-risk populations (eg, first-degree relatives, those with specific human leukocyte antigen variant genotypes, and those with certain chromosomal disorders, such as Down syndrome and Turner syndrome).2-6 However, despite the increasing prevalence of CD, most patients remain undiagnosed.1
The diagnosis of CD in adults is typically made with elevated serum tTG-IgA and
STUDY SUMMARY
tTG-IgA titers were highly predictive of CD in 3 distinct cohorts
This 2021 hybrid prospective/retrospective study with 3 distinct cohorts aimed to assess the utility of serum tTG-IgA titers compared to traditional EGD with duodenal biopsy for the diagnosis of CD in adult participants (defined as ≥ 16 years of age). A serum tTG-IgA titer ≥ 10 times the ULN was set as the minimal cutoff value, and standardized duodenal biopsy sampling and evaluation for histologic mucosal changes consistent with
Continue to: Cohort 1 was a...
Cohort 1 was a prospective analysis of adults (N = 740) considered to have a high suspicion for CD, recruited from a single CD subspecialty clinic in the United Kingdom. Patients with a previous diagnosis of CD, those adhering to a gluten-free diet, and those with IgA deficiency were excluded. Study patients had tTG-IgA titers drawn and, within 6 weeks, underwent endoscopy with ≥ 1 biopsy from the duodenal bulb and/or the second part of the duodenum. The PPV of tTG-IgA titers ≥ 10 times the ULN in patients with biopsy-proven CD was 98.7% (95% CI, 97%-99.4%).
Cohort 2 was a retrospective analysis of adult patients (N = 532) considered to have low suspicion for CD. These patients were referred for endoscopy for generalized GI complaints in the same hospital as Cohort 1, but not the subspecialty clinic. Exclusion criteria and timing of IgA titers and endoscopy were identical to those of Cohort 1. The PPV of tTG-IgA titers ≥ 10 times the ULN in patients with biopsy-proven CD was 100%.
Cohort 3 (which included patients in 8 countries) was a retrospective analysis of the performance of multiple assays to enhance the validity of this approach in a wide range of settings. Adult patients (N = 145) with tTG-IgA serology positive for celiac who then underwent endoscopy with 4 to 6 duodenal biopsy samples were included in this analysis. Eleven distinct laboratories performed the tTG-IgA assay. The PPV of tTG-IgA titers ≥ 10 times the ULN in patients with biopsy-proven CD was 95.2% (95% CI, 84.6%-98.6%).
In total, this study included 1417 adult patients; 431 (30%) had tTG-IgA titers ≥ 10 times the ULN. Of those patients, 424 (98%) had histopathologic findings on duodenal biopsy consistent with CD.
Of note, there was no standardization as to the assays used for the tTG-IgA titers: Cohort 1 used 2 different manufacturers’ assays, Cohort 2 used 1 assay, and Cohort 3 used 5 assays. Regardless, the “≥ 10 times the ULN” calculation was based on each manufacturer’s published assay ranges. The lack of assay standardization did create variance in false-positive rates, however: Across all 3 cohorts, the false-positive rate for trusting the “≥ 10 times the ULN” threshold as the sole marker for CD in adults increased from 1% (Cohorts 1 and 2) to 5% (all 3 cohorts).
Continue to: WHAT'S NEW
WHAT’S NEW
Less invasive, less costly diagnosis of celiac disease in adults
In adults with symptoms suggestive of CD, the diagnosis can be made with a high level of certainty if a serum tTG-IgA titer is ≥ 10 times the ULN. Through informed, shared decision making in the presence of such a finding, patients may accept a serologic diagnosis and forgo an invasive EGD with biopsy and its inherent costs and risks. Indeed, if the majority of patients with CD are undiagnosed or underdiagnosed, and there exists a minimally invasive blood test that is highly cost effective in the absence of “red flags,” the overall benefit of this path could be substantial.
CAVEATS
“No biopsy” does not mean no risk/benefit discussion
While the PPVs are quite high, the negative predictive value varied greatly: 13%, 98%, and 10% for Cohorts 1, 2, and 3, respectively. Therefore, although serum tTG-IgA titers ≥ 10 times the ULN are useful for diagnosis, a negative result (serum tTG-IgA titers < 10 times the ULN) should not be used to rule out CD, and other testing should be pursued.
Additionally (although rare), patients with CD who have IgA deficiency may obtain false-negative results using the tTG-IgA ≥ 10 times the ULN diagnostic criterion.7,8
Also, both Cohorts 1 and 2 took place in general or subspecialty GI clinics (Cohort 3’s site types were not specified). However, the objective interpretation of tTG-IgA serology means it could be considered as an additional diagnostic tool for primary care physicians, as well.
Finally, if a primary care physician and their patient decide to go the “no-biopsy” route, it should be with a full discussion of the possible risks and benefits of not pursuing EGD. If there are any potential “red flag” symptoms suggesting the possibility of a more concerning differential diagnosis, EGD evaluation should still be pursued. Such symptoms might include (but not be limited to) chronic dyspepsia, dysphagia, weight loss, and unexplained anemia.7
Continue to: CHALLENGES TO IMPLEMENTATION
CHALLENGES TO IMPLEMENTATION
Diagnostic guidelines still favor EGD with biopsy for adults
The 2013 American College of Gastroenterology guidelines support the use of EGD and duodenal biopsy to diagnose CD in both low- and high-risk patients, regardless of serologic findings.7 In a 2019 Clinical Practice Update, the American Gastrointestinal Association (AGA) stated that when tTG-IgA titers are ≥ 10 times the ULN and EMAs are positive, the PPV is “virtually 100%” for CD. Yet they still state that in this scenario “EGD and duodenal biopsies may then be performed for purposes of differential diagnosis.”8 Furthermore, the AGA does not discuss informed and shared decision making with patients for the option of a “no-biopsy” diagnosis.8
Additionally, there may be challenges in finding commercial laboratories that report reference ranges with a clear ULN. Although costs for the serum tTG-IgA assay vary, they are less expensive than endoscopy with biopsy and histopathologic examination, and therefore may present less of a financial barrier.
ILLUSTRATIVE CASE
A 43-year-old woman presents to the clinic with diffuse, intermittent abdominal discomfort, bloating, and diarrhea that has slowly but steadily worsened over the past few years to now-daily symptoms. She states her overall health is otherwise good. Her review of systems is pertinent only for 8 lbs of unintentional weight loss over the past year and increased fatigue. She takes no supplements or routine over-the-counter or prescription medications, except for low-dose combination oral contraceptives, and is unaware of any family history of gastrointestinal (GI) diseases. She does not drink or smoke. She is up to date with immunizations and with cervical and breast cancer screening. Her body mass index is 23, her vital signs are within normal limits, and her physical exam is normal except for mild, diffuse abdominal tenderness without any masses, organomegaly, or peritoneal signs.
Her diagnostic work-up includes a complete metabolic panel, magnesium level, complete blood count, thyroid-stimulating hormone measurement, cytomegalovirus IgG and IgM serology, and stool studies for fecal leukocytes, ova and parasites, and fecal fat, in addition to a kidney, ureter, and bladder noncontrast computed tomography scan. All diagnostic testing is negative except for slightly elevated fecal fat, thereby decreasing the likelihood of infection, thyroid disorder, electrolyte abnormalities, or malignancy as a source of her symptoms.
She says that based on her online searches, her symptoms seem consistent with CD—with which you concur. However, she is fearful of an endoscopic procedure and asks if there is any other way to diagnose CD.
CD is an immune-mediated disorder in genetically susceptible people that is triggered by dietary gluten, causing damage to the small intestine.1-6 The estimated worldwide prevalence of CD is approximately 1%, with greater prevalence in females.1-6 A strong genetic predisposition also has been noted: prevalence among first-degree relatives is 10% to 44%.2,3,6 Although CD can be diagnosed at any age, in the United States the mean age at diagnosis is in the fifth decade of life.6
The incidence of CD is on the rise due to true increases in disease incidence and prevalence, increased detection through better diagnostic tools, and increased screening of at-risk populations (eg, first-degree relatives, those with specific human leukocyte antigen variant genotypes, and those with certain chromosomal disorders, such as Down syndrome and Turner syndrome).2-6 However, despite the increasing prevalence of CD, most patients remain undiagnosed.1
The diagnosis of CD in adults is typically made with elevated serum tTG-IgA and
STUDY SUMMARY
tTG-IgA titers were highly predictive of CD in 3 distinct cohorts
This 2021 hybrid prospective/retrospective study with 3 distinct cohorts aimed to assess the utility of serum tTG-IgA titers compared to traditional EGD with duodenal biopsy for the diagnosis of CD in adult participants (defined as ≥ 16 years of age). A serum tTG-IgA titer ≥ 10 times the ULN was set as the minimal cutoff value, and standardized duodenal biopsy sampling and evaluation for histologic mucosal changes consistent with
Continue to: Cohort 1 was a...
Cohort 1 was a prospective analysis of adults (N = 740) considered to have a high suspicion for CD, recruited from a single CD subspecialty clinic in the United Kingdom. Patients with a previous diagnosis of CD, those adhering to a gluten-free diet, and those with IgA deficiency were excluded. Study patients had tTG-IgA titers drawn and, within 6 weeks, underwent endoscopy with ≥ 1 biopsy from the duodenal bulb and/or the second part of the duodenum. The PPV of tTG-IgA titers ≥ 10 times the ULN in patients with biopsy-proven CD was 98.7% (95% CI, 97%-99.4%).
Cohort 2 was a retrospective analysis of adult patients (N = 532) considered to have low suspicion for CD. These patients were referred for endoscopy for generalized GI complaints in the same hospital as Cohort 1, but not the subspecialty clinic. Exclusion criteria and timing of IgA titers and endoscopy were identical to those of Cohort 1. The PPV of tTG-IgA titers ≥ 10 times the ULN in patients with biopsy-proven CD was 100%.
Cohort 3 (which included patients in 8 countries) was a retrospective analysis of the performance of multiple assays to enhance the validity of this approach in a wide range of settings. Adult patients (N = 145) with tTG-IgA serology positive for celiac who then underwent endoscopy with 4 to 6 duodenal biopsy samples were included in this analysis. Eleven distinct laboratories performed the tTG-IgA assay. The PPV of tTG-IgA titers ≥ 10 times the ULN in patients with biopsy-proven CD was 95.2% (95% CI, 84.6%-98.6%).
In total, this study included 1417 adult patients; 431 (30%) had tTG-IgA titers ≥ 10 times the ULN. Of those patients, 424 (98%) had histopathologic findings on duodenal biopsy consistent with CD.
Of note, there was no standardization as to the assays used for the tTG-IgA titers: Cohort 1 used 2 different manufacturers’ assays, Cohort 2 used 1 assay, and Cohort 3 used 5 assays. Regardless, the “≥ 10 times the ULN” calculation was based on each manufacturer’s published assay ranges. The lack of assay standardization did create variance in false-positive rates, however: Across all 3 cohorts, the false-positive rate for trusting the “≥ 10 times the ULN” threshold as the sole marker for CD in adults increased from 1% (Cohorts 1 and 2) to 5% (all 3 cohorts).
Continue to: WHAT'S NEW
WHAT’S NEW
Less invasive, less costly diagnosis of celiac disease in adults
In adults with symptoms suggestive of CD, the diagnosis can be made with a high level of certainty if a serum tTG-IgA titer is ≥ 10 times the ULN. Through informed, shared decision making in the presence of such a finding, patients may accept a serologic diagnosis and forgo an invasive EGD with biopsy and its inherent costs and risks. Indeed, if the majority of patients with CD are undiagnosed or underdiagnosed, and there exists a minimally invasive blood test that is highly cost effective in the absence of “red flags,” the overall benefit of this path could be substantial.
CAVEATS
“No biopsy” does not mean no risk/benefit discussion
While the PPVs are quite high, the negative predictive value varied greatly: 13%, 98%, and 10% for Cohorts 1, 2, and 3, respectively. Therefore, although serum tTG-IgA titers ≥ 10 times the ULN are useful for diagnosis, a negative result (serum tTG-IgA titers < 10 times the ULN) should not be used to rule out CD, and other testing should be pursued.
Additionally (although rare), patients with CD who have IgA deficiency may obtain false-negative results using the tTG-IgA ≥ 10 times the ULN diagnostic criterion.7,8
Also, both Cohorts 1 and 2 took place in general or subspecialty GI clinics (Cohort 3’s site types were not specified). However, the objective interpretation of tTG-IgA serology means it could be considered as an additional diagnostic tool for primary care physicians, as well.
Finally, if a primary care physician and their patient decide to go the “no-biopsy” route, it should be with a full discussion of the possible risks and benefits of not pursuing EGD. If there are any potential “red flag” symptoms suggesting the possibility of a more concerning differential diagnosis, EGD evaluation should still be pursued. Such symptoms might include (but not be limited to) chronic dyspepsia, dysphagia, weight loss, and unexplained anemia.7
Continue to: CHALLENGES TO IMPLEMENTATION
CHALLENGES TO IMPLEMENTATION
Diagnostic guidelines still favor EGD with biopsy for adults
The 2013 American College of Gastroenterology guidelines support the use of EGD and duodenal biopsy to diagnose CD in both low- and high-risk patients, regardless of serologic findings.7 In a 2019 Clinical Practice Update, the American Gastrointestinal Association (AGA) stated that when tTG-IgA titers are ≥ 10 times the ULN and EMAs are positive, the PPV is “virtually 100%” for CD. Yet they still state that in this scenario “EGD and duodenal biopsies may then be performed for purposes of differential diagnosis.”8 Furthermore, the AGA does not discuss informed and shared decision making with patients for the option of a “no-biopsy” diagnosis.8
Additionally, there may be challenges in finding commercial laboratories that report reference ranges with a clear ULN. Although costs for the serum tTG-IgA assay vary, they are less expensive than endoscopy with biopsy and histopathologic examination, and therefore may present less of a financial barrier.
1. Penny HA, Raju SA, Lau MS, et al. Accuracy of a no-biopsy approach for the diagnosis of coeliac disease across different adult cohorts. Gut. 2021;70:876-883. doi: 10.1136/gutjnl-2020-320913
2. Al-Toma A, Volta U, Auricchio R, et al. European Society for the Study of Coeliac Disease (ESsCD) guideline for coeliac disease and other gluten-related disorders. United European Gastroenterol J. 2019;7:583-613. doi: 10.1177/2050640619844125
3. Caio G, Volta U, Sapone A, et al. Celiac disease: a comprehensive current review. BMC Med. 2019;17:142. doi: 10.1186/s12916-019-1380-z
4. Lebwohl B, Rubio-Tapia A. Epidemiology, presentation, and diagnosis of celiac disease. Gastroenterology. 2021;160:63-75. doi: 10.1053/j.gastro.2020.06.098
5. Lebwohl B, Sanders DS, Green PHR. Coeliac disease. Lancet. 2018;391:70-81. doi: 10.1016/S0140-6736(17)31796-8
6. Rubin JE, Crowe SE. Celiac disease. Ann Intern Med. 2020;172:ITC1-ITC16. doi: 10.7326/AITC202001070
7. Rubio-Tapia A, Hill ID, Kelly CP, et al; American College of Gastroenterology. ACG clinical guidelines: diagnosis and management of celiac disease. Am J Gastroenterol. 2013;108:656-676; quiz 677. doi: 10.1038/ajg.2013.79
8. Husby S, Murray JA, Katzka DA. AGA clinical practice update on diagnosis and monitoring of celiac disease—changing utility of serology and histologic measures: expert review. Gastroenterology. 2019;156:885-889. doi: 10.1053/j.gastro.2018.12.010
9. Husby S, Koletzko S, Korponay-Szabó I, et al. European Society Paediatric Gastroenterology, Hepatology and Nutrition guidelines for diagnosing coeliac disease 2020. J Pediatr Gastroenterol Nutr. 2020;70:141-156. doi: 10.1097/MPG.0000000000002497
1. Penny HA, Raju SA, Lau MS, et al. Accuracy of a no-biopsy approach for the diagnosis of coeliac disease across different adult cohorts. Gut. 2021;70:876-883. doi: 10.1136/gutjnl-2020-320913
2. Al-Toma A, Volta U, Auricchio R, et al. European Society for the Study of Coeliac Disease (ESsCD) guideline for coeliac disease and other gluten-related disorders. United European Gastroenterol J. 2019;7:583-613. doi: 10.1177/2050640619844125
3. Caio G, Volta U, Sapone A, et al. Celiac disease: a comprehensive current review. BMC Med. 2019;17:142. doi: 10.1186/s12916-019-1380-z
4. Lebwohl B, Rubio-Tapia A. Epidemiology, presentation, and diagnosis of celiac disease. Gastroenterology. 2021;160:63-75. doi: 10.1053/j.gastro.2020.06.098
5. Lebwohl B, Sanders DS, Green PHR. Coeliac disease. Lancet. 2018;391:70-81. doi: 10.1016/S0140-6736(17)31796-8
6. Rubin JE, Crowe SE. Celiac disease. Ann Intern Med. 2020;172:ITC1-ITC16. doi: 10.7326/AITC202001070
7. Rubio-Tapia A, Hill ID, Kelly CP, et al; American College of Gastroenterology. ACG clinical guidelines: diagnosis and management of celiac disease. Am J Gastroenterol. 2013;108:656-676; quiz 677. doi: 10.1038/ajg.2013.79
8. Husby S, Murray JA, Katzka DA. AGA clinical practice update on diagnosis and monitoring of celiac disease—changing utility of serology and histologic measures: expert review. Gastroenterology. 2019;156:885-889. doi: 10.1053/j.gastro.2018.12.010
9. Husby S, Koletzko S, Korponay-Szabó I, et al. European Society Paediatric Gastroenterology, Hepatology and Nutrition guidelines for diagnosing coeliac disease 2020. J Pediatr Gastroenterol Nutr. 2020;70:141-156. doi: 10.1097/MPG.0000000000002497
PRACTICE CHANGER
Consider a “no-biopsy” approach by evaluating serum immunoglobulin (Ig) A anti-tissue transglutaminase (tTG-IgA) antibody titers in adult patients who present with symptoms concerning for celiac disease (CD). An increase of ≥ 10 times the upper limit of normal (ULN) for tTG-IgA has a positive predictive value (PPV) of ≥ 95% for diagnosing CD when compared with esophagogastroduodenoscopy (EGD) with duodenal biopsy—the current gold standard.
STRENGTH OF RECOMMENDATION
A: Consistent findings from 3 good-quality diagnostic cohorts presented in a single study.1
Penny HA, Raju SA, Lau MS, et al. Accuracy of a no-biopsy approach for the diagnosis of coeliac disease across different adult cohorts. Gut. 2021;70:876-883. doi: 10.1136/gutjnl-2020-320913